Sie sind auf Seite 1von 89

Diya Coaching Centre for Nurses

One candle wipes out darkness…….

1. The nurse administrated a dose of morphine sulphate as prescribed to a patent who is in


PACU. The patient appears to be resting comfortably the respiratory rate is 8 and O2
saturation is 86% via O2 cannula. The Nurse should immediately administer
A. Flumazenil(Romazicon)
B. Midazolum (versed)
C. Nalaxone
D. Ondansetron
2. A patient schedule for a major surgery in one hour is very nervous and upset. Which of the
following medications would the nurse administrator to relax this patient?
A. Meperidine hydrochloride (Demerol)
B. Scopolumine (Tranderm-scop)
C. Pentobarbital sodium (Nembutal sodium)
D. Trazodone Hydrochloride (Trazdone)
3. A patient with poor wound healing and poor appetite has an order to begin total parentral
nutrition(TPN). Waiting for the TPN solution to arrive from the pharmacy the Nurse should
obtain
A. A pair of sterile glove
B. An infusion Pump
C. IV tubing with amicro-drip chamber
D Providone- iodine (Betadine) swabs.
4. A nurse is caring for a parent receiving TPN The patient reports the sudden onset of feeling
short of breath and anxious. The nurse hears crackles in bilateral lower lobes of the lungs
and the patients O2 saturation is 90% on room air. The nurse nest immediately.
A. Turn off the TPN
B. Notify the physician
C Assess the patient’s capillary blood glucose level
D. Attempt to suction the patient’s airway.
5. A nurse just started total parental nutrition as prescribed for a patient with severe dysphagia,
low pre albumin levels. In one to hours the nurse should .................. assessing the patient.
A. Blood glucose level
B. Weight
C. Liver
D O2 saturation
6. The nurse is planning care for several children who were admitted during the shift. Daily weight
should be of the plan of care for the child who is receiving
A. TPN
B. Supplement O2
C. Intravenous anti-ineffective
D. Chest physiotherapy
7. The nurse is caring for a 4 year old.... a diagnosis of cystic fibrosis and pneumonia. The child is
feeling better on the 3rd day of the hospitalized and “wants to play”. What would be the best choice
of entertainment?
A. Blowing bubbles
Compiled by : V.Devi | 9176868900 Page 1
Diya Coaching Centre for Nurses
One candle wipes out darkness…….

B. Looking at picture books.


C. Watching videos
D. Riding in a wagon
8. A nurse in caring for an 8 year old male with cystic fibrosis, based on the disease condition what
nursing intervention should the nurse expect to perform?
A Restrict sodium and fluid intake
B.Give anti-diarrheal medication
C. Discourage coughing after postural drainage pancreatic enzy
D.Administrater pancreatic enzymes with each meal
9. A nurse is caring for a child with a diagnosis of cystic fibrosis and pneumonia. The plan of care
includes Nebulizer Rx and chest physiotherapy. The nurse should perform chest physiotherapy
A. Continusly during nebulizer Rx
B. Prior to the nebulizer Rx
C. After the Nebulizer Rx.
D. Intermittently during nebulizer Rx
10. When conducting discharge teaching for the parent of child newly diagnosed with cystic fibrosis.
Which of the following statement by the parent indicates the need for further teaching?
A. weekly weights help evaluate effectiveness of nutritional intervention
B. Weekly weights help the doctor know if my child is absorbing nutrients
C. Weekly weights re-assure that my child recovery is progressing
D. Weekly weights help the doctor know if my child needs additional enzymes
11. While caring for a pt with an ileostomy, the nurse would expect the ostomy to be located in
which quadrant of abdomen?
A. Right lower
B. Left lower
C. Left Upper
D. Right Upper
12. A patient has been assessed and found to have a severe dysphagia and will need long term
nutritional support. Which one of the following types of feeding would most likely to be beneficial
for the patient?
A. Gastrostomy
B. Parenteral
C. Nasogastric
D. Nasoduodenal
13. A Surgeon instructs a nurse to serve as a witness to an elderly patient informed consent for
surgery. During explanations to the pt it becomes clear and the patient is confused and does not
understand the procedure but reluctantly signs the consent form. The Nurse should
A. Sign the form as a witness making a notion that pt did not appear to understand
B. Not sign the form as a witness and notify the nurse supervisor
C. Not sign the form and tell surgeon that the pt dosen’t understand the procedure.
D. Sign the form and tell surgeon that the pt dosen’t understand the procedure.

Compiled by : V.Devi | 9176868900 Page 2


Diya Coaching Centre for Nurses
One candle wipes out darkness…….

14. The nurse evaluating the pt with an end stage COPD. The pt has not achieved any of the goals in
the plan of care. The spouse reports concerns about the pt’s mood and increased dependency. What
action should the nurse take first?
A. Continue care plan for one more month
B. Refer pt to psychiatric services
C. Collaborate with pt and spouse to revise the care plan
D. Revise the care plan based on spouses input
15. A home care pt with COPD reports an upset stomach. The Pt is taking
.......................................................................................................................................
A. Theo-dur at empty stomach
B. Theo-dur and azmacort at the same time
C Theodur and azmacort 12 hour apart
D. Theodur at night
16. When giving post operative discharge instructions to a patient who had abdominal surgery all
of the following regarding would healing are true except?
A.Bathing to soak abdomen is preffered over showing
B. Avoid tight belts and cloths which seems that may rub the wound
C.Pain medication may effect ability to drive
D. Irregular bowel movements can be expected
17. A nurse giving post operative discharge instruction to a patient who had abdominal surgery.
When teaching the patient about wound healing all of the following are true except?
A.Wound may feel tightly or itching as healing occurs
B. Scabs promote infection of the new skin underneath them
C. Numbmness or a slight pulling sensation is normal
D. Wound should not have any drainage
18. A 12 year old child who has been diagnosed with insulin dependent mellitus(IDDM) since age 3
comes to the clinic for a routine visit. The patient has begin to self manage care with parental
supervision. The patient injects 28 units of NPH insulin every morning and 8 units at bed time. The
patient checks blood sugar 4 times every day The patient’s weight is stable and diet is unchanged.
However the patient report several hypoglycaemic reactions every week. The nurse knows the most
weekly cause is that:-
A. The patient is not eating the adequate number of calories reported
B. The dosage of insulin may need to be decresed as the patient continues grow
C.There may be changes in exercise, stress level or the beginning of growth spurt
D. The patient may not be competent in techniques of drawing up and injection insulin.
19. A nurse visits a patient at home who dosent understand how to take a new prescribed
medication. The prescription reads 5ml PO tds P.C meals The nurse explains to the patient that the
correct way to take the medication is;
A. 1 table spoon by mouth3 times a day before meals
B.1 teaspoon by mouth 3 times a day after meals
C.1 teaspoon by mouth 3 times a day before meals
D. 1 tablespoon by mouth 3 times a day after meals

Compiled by : V.Devi | 9176868900 Page 3


Diya Coaching Centre for Nurses
One candle wipes out darkness…….

20. The nurse is caring for a patient who had major abdominal surgery under GA hours ago. An
appropriate goal for the patient includes
A. Having minimal fine crackles in the base of lungs ..............................................................
.........................................................................
21. While caring for a child with a ventriculo peritoneal shunt revision the nurse finds the patient
lying with the head and feet flexed back. The nurse should call for help and prepare for
A. Spinal tap
B. Shunt culture
C. Electro cardiogram
D.Ventricular tap
22. A patient undergoing treatment for cancer with bone metastasis is experiencing severe pain.
Which of the following treatment would the nurse most likely expect to improve the patient’s pain
control?
A Adjuvant radiation therapy
B. Palliative radiation therapy
C. Curative radiation therapy
D. Ventricular Tap
23. During surgery requiring general anesthesia the patient’s heart stops and carotid pulse is not
palpated How many compressions per minutes should be administrated?
A.50
B.60
C.80
D.100
24. When teaching a community class on cerebrovascular accidents, which of the following should
partiapants of the class know at the completion of the class?
A. Muscle and ligament damage is not reversible
B. Expressive aphasia is resolved by voice rest
C. There is a risk for mood disorders such as depression
D. Liquids should be consumed at the same time as solids foods
25. A community health care nurse visits a pt who had a CVA. The pt is at risk for deficient volume
due to voluntary reduction fluid intake to avoid the use of bathroom The Nurse educates the pt on
the importance of drinking fluids and maintaining hydration. Which indicates efficient nursing
intervention?
A. Amber color urine
B. Respiration of 35
C. Tachycardia
D. Moist mucous membrane
26. A home health nurse is visiting a pt following a CVA The pt is having trouble sleeping and is
feeling sad. The pt’s spouse tells the nurse that the patient is not eating much and often cries when
no one is watching. Which of the following would be the nurse’s most likely intervention?
A. Assess for changes in congnitive abilities
B. Complete a depression index

Compiled by : V.Devi | 9176868900 Page 4


Diya Coaching Centre for Nurses
One candle wipes out darkness…….

C. Strengthen family coping methods


D. Screen for pain
27. A home health Nurse is visiting a pt who recently suffered a CVA the nurse would most likely
implement which of the following intervention to prevent muscle and ligament deformities?
A. Daily moist heat and isometric exercises
B. Daily balance training and routine medications for pain
C. Instruct the pt to use non-affected side to perform activities of daily living
D. Daily range of motion exercises
28. A nurse assigned to do a home visit for an 81 yr old pt. The pt lives at home with an adult care
taken and is completely bed-bound following a CVA, 2weeks ago. In planning care giver education,
the nurse should be prepared to instruct the care taker in
A. How to select Nursing home for pt
B. Performing passive range of motion exercises
C. The importance of avoiding viscous drinks
D. Forming a local chapter of care giver support group
29. A home care nurse makes a follow-up visit to a pt who recently suffered a CVA. The pt is
mobile and able to perform activities of daily living. However the pt has not sleeping and has lost
weight due to lack of appetite. The pt also feels overwhelmed with sadness Which of the following
is the most appropriate evaluation
A. Pt’s progress is as expected and no further intervention is necessary
B. Pt needs referral to nutritionist
C. Pt needs intervention for depression
D. Pt needs sleeping medication
30. A patient admitted with CVA is unable to chew or swallow food with risk for aspiration. The
nurse would anticipate receiving which of the following order for the patient?
A. Give no food by mouth and start IV hydration
B. Start a pureed diet with thickened liquids
C. Refer the patient to psychiatrist for depression related to the CVA
D. Refer the patient to physical therapy for muscle strengthening
31. While the nurse is administering a large volume enema, the patient complaints of cramps the
nurse should
A. increase the flow rate
B. Lower the fluid container
C. Elevate the head of bed
D. Gently message the abdomen
32. A home health nurse has entered a home to complete an admission assessment on a pt who has
a methicillin resistant staphylococcus aureus (MRSA) UTI. The pt will receive IV anti-infective via a
peripherally inserted central catheter for 3 weeks. Which of the following actions should the nurse
take first?
A. Shake the patient’s hand
B. Place the nursing supply bag on a clean dry surface
C. Obtain pt’s written consent for home health care

Compiled by : V.Devi | 9176868900 Page 5


Diya Coaching Centre for Nurses
One candle wipes out darkness…….

D. Perform hand hygiene per the agency protocol.


33. A home health nurse is teaching a family member about the care of pt’s peripherally inserted
central catheter(PICC).Which of the following would be appropriate for the nurse to make?
A. Place the used IV tubing in a leak proof container.
B. You will need to put on a disposable face mask before you connect the intraveinous tubing
to the port of the PICC.
C. The port of PIC catheter will need to be cleansed the povidone-iodine(Betadine) after
the infusion is completed.
D. The empty medication container can be place in the same container as your household
refuses.
34. A pt had a craniotomy with resection of a non-malignant neoplasm for the temporal lobe. The
pt’s vital signs are within the baseline normal range. The nurse observes that the pt has developed
bilateral periorbital edema. Which of the following actions would be appropriate for the nurse to
take?
A. Apply cold compress to the pt’s eye.
B. Apply warm compress to the pt’s eye.
C. Elevate the head of the pt’s bed to 60 degree.
D. Elevate the head of the pt’s bed to 45 degree.
35. To decrease the incidence of aspiration of gastric contents in a child hospitalized with severe
burns, the nurse should position the head.
A. Flat expect during meals.
B.Elevate 30-45 degree during meals.
C.Elevate 15-30 degree for 12 hours after meals.
D.Elevate 45 degree all times.
36. A home health nurse visit a patient with diabetes and primary open angle glaucoma. The
patient take metformin (Glyciphage) 500 mg OD for diabetes &timolol ophthalmic solution BD in
each eye for glaucoma. Which of the following elevations indicate that the patient is compliant with
glaucoma management?.
A. Patient has not been taking glyciphage.
B. Patient has tearing of the eye.
C. Patient has not refilled prescription for timolol in 3 months.
D. Patient has yellow discharge from the eyes.
37. A pt is having difficulty with cognitive abilities after a stroke. What part of brain is affected?
A. Mid brain
B. Cerebrum
C. Medulla oblongata
D. Cerebellum
38. A 16 year old patient present to the clinic requesting birth control; with the diagnosis of health
seeking behaviour the best goals have the pt:
A. Verbalising , understanding of safe sex production and following safe sexual practices in
all encounters.

Compiled by : V.Devi | 9176868900 Page 6


Diya Coaching Centre for Nurses
One candle wipes out darkness…….

B Not engaging in sexual encounters until she is 18 yrs of old and maintaining a healthy life
style.
C. Recognizing the sign of pregnancy, the symptom of STD
D. Understanding safe sexual practice and use of condom to prevent pregnancy and STD
39. A nurse plan to teach a group of 20-25 years old women about oral contraceptives. The nurse
should instruct that oral contraceptives may;
A. Increase the risk of pelvic inflammatory disease
B. Cause acne to worsen
C. Decrease the risk of breast and cervical cancer
D. Decrease the risk of endometriosis
40. Following lumbar surgery a pt has a 4 mm surgical incision The incision is clean and the edges
are well appropriate. The type of tissue healing is classified as which of the following?
A. Primary intention
B. Secondary intention
C. Tertiary intention
D. Superficial epidermal
41. A shrinkage device is applied after surgery for amputation of the leg. The goal of shrinking
device is to form the residual limb into what shape?
A. Cone
B. Oval
C. Mushroom
D Cylinder c blunt end
42. A pt pulmonary embolus and a nurse diagnosis of impared gas exchange has an order to obtain
ABG. The first intervention by the nurse is to;
A. Perform Allen test
B. Explain the procedure
C. Gather the equipment
D. Document the procedure
43. A pt is diagnosed with pulmonary hypertension. Which of the following nursing diagnosis
should be the priority?
A. Impaired gas exchange related to altered blood flow secondary to pulmonary capillary
constriction
B. Fatigue related to hypoxia
C. Anxiety related to illness and loss of control
D. Activity intolerance related to imbalance between O2 supply and demand related to right
and left ventricular failure
44. A pt who had abdominal surgery in PACU which of the following nursing diagnosis take
priority?
A. Disturbed sleep pattern
B Acute pain
C. Risk for infection
D. Infective airway clearence

Compiled by : V.Devi | 9176868900 Page 7


Diya Coaching Centre for Nurses
One candle wipes out darkness…….

45. While caring for a pt in the PACU a nurse observes the onset of rapid breathing and cyanosis,
and narrowing blood pressure. The nurse’s should plan to
A. Administer bolus glucose
B. Suction airway
C. Turn the pt to right side
D. Administer IV fluids
46 . While caring for a pt in the PACU , who has developed hypovolemic shock, a nurse should
position the pt.
A. Flat with legs elevated
B. In trendelenberg position
C. With the head of bed elevated
D. Completely flat
47. A pt vitrectomy and is about to be transported to the post anesthesia care unit(PACU) The pt
should be placed in which of the following positions before transport to PACU?
A Semi fowler’s
B. prone
C Dorsal recumbent
D. Sims
48. While caring for a pt in PACU the nurse plan to keep pt warm. What is the important reason for
the action is?
A. To preserve nutritional stores
B. to prevent cutaneous vessel dilation
C. To decrease pt anxiety
D. to lower risk of infection resulting from chilling
49 . A pt had a total abdominal hysterectomy 2 days-ago and has not been out of bed yet. The pt
complaining left leg pain and swelling. What should the nurse do first?
A. Gently massage the pts leg
B. Assess the pt for Homan’s sign
C. Assess the pt to reflex the left knee and lip
D. Instruct the pt to reflex the left knee lip
50. To minimize a toddler from scratching and picking at a healing skin graft site, the nurse should
utilize
A. Mild sedative
B. hand mittens
C. Punishment
D Distraction
51. The nurse teaching the mother of a 3 months old infant about bottle feeding. Which statement
indicates the mother understands of appropriate procedure?
A. It is Ok to prop the bottle on a pillow
B. I can deed my whole milk
C. I should hold my baby in slightly reclined position , close to my body
D. I should warm the bottles in the microwave if they come out of the refrigeration

Compiled by : V.Devi | 9176868900 Page 8


Diya Coaching Centre for Nurses
One candle wipes out darkness…….

52. A 9 month old child who has had four ear infections in the past 6 months is being discharged
which statement by the parent indicates the need for further discharge teaching?
A. I should never put my baby to bed with bottle
B. My child should not use a pacifier ago 6 months
C. My child should drinks his bottle while laying flat in my lap
D. My child should not be around people who smoke
53. A patient complaint of severe menstrual cramping, bleeding is not unusually heavy. And the
patient has no uterine disorders. Which of the following interventions should the nurse anticipate
the doctor will order to promote comfort?
A. Acetaminophen
B. Strict bed rest
C. Heating pad to the back of neck
D. Ibuprofen (motrin)
54. During pre operative preparation of a patient for amputation of the left leg The nurse has
primary responsibility for
A. witnessing the patient signature on the consent form
B Explaining the procedure to the patient
C.Explaining the risk of the surgery to the patient
D.Making appropriate incision lines on the leg
55. A 52 years old is admitted to the nursing unit from the physician office with a diagnosis of acute
cholecystitis. Physician order on admission include monitor vital signs for every 4 hrs. IV of ringers
lactate 125ml/hr. 1500 calorie, low fat liquid diet, morphine sulphate 2 mg IV every2hours as need
for pain, notify physician for sudden increase in frequency or intensity of pain, promethazine
12.5mg IV every 4 hours as needed for nausea or vomiting. Which of them following should the
nurse plan to do FIRST?
A. Remove any high foods from the patients room
B. notify the dietitian of the diet order
C. Obtain venous access and start Ringer’s lactate infusion
D Obtain an emisis basin and clean linen from the bed side.
56. A patient brings a 10 month old infant into the department saying “my baby put a button in her
mouth and now she is not breathing. After the nurse determines the infant is not breathing. What
should the nurse do NEXT?
A. Perform the Heimlich maneuver
B. Initiate cardio pulmonary resuscitation (CPR)
C. Administer 4 back blows
D. Admistrater 4 thrusts midline on the patient back
57. An infant arrives in the emergency department not breathing and does have a pulse When
starting cardiopulmonary resuscitation ,where is the correct place to assess for a pulse in this
patient?
A. Carotid
B. Radial
C.Brachial

Compiled by : V.Devi | 9176868900 Page 9


Diya Coaching Centre for Nurses
One candle wipes out darkness…….

D. Temporal
58. A 5 year old patient who underwent abdominal surgery suffers from deficient fluid volume
related to nothing by month (NPO) status. Intravenous fluid therapy is given for hydration Which
of the following indicates that the treatment is effective?
A. Urinary output of 15ml/hr
B. Respiratory rate 35
C. Heart rate 100
D. Good skin turgor
59. A 7 year child is brought to the physician’s office due to sudden onset of bright redness on the
checks. The nurse observe that the child has a temperature of 380 c(1040F) with this the nurse suspect
that the most like diagnosis would be
A. Fifthe disease
B. Rota virus
C. Roscola Infantum
D. Monitor patient for shock
60. What are the common ventilator complication?
A. Hypotension
B. Hyperventilation
C. Bucking with Ventilator
D. VAP
61. Expain the first management for digoxin toxicity?
A. Administer digibind
B.With hold the drug
C. Administer beta blockers
D. Monitor ECG
62. A patient came with burn on chest& arm which artery can be used for ABG?
A. Radial
B. Brachial
C. Femoral
D. Carotid
63. What is barrel chest?
A. AP=transverse diameter
B AP>TD
C.AP<TD
D.AP>TD>
64. What is the care of a patient with diverticulosis?
A.Provide high residue diet
B. Provide low residue diet
C.provide high residue diet without seed
D. Low calorie diet

65. What all are the instruiction given to a patient using cane?

Compiled by : V.Devi | 9176868900 Page 10


Diya Coaching Centre for Nurses
One candle wipes out darkness…….

A Palce the cane close to the body


B. Place the cane on the affected side
C. Place the cane on the unaffected side
D.Hold the cane with the hand in straight manner
66. Why gluteal muscle is not used in infants for IM injection?
A. It have high vascularity
B. It is immature
C. High chance for muscle atrophy
D. High chance for abscess formation
67 . Patient receiving penicillin suddenly develops red rashes. First action?
A. Administer glucocorticoids
B. Administer adrenaline
C. Withhold the drug
D. Administer antihistamines.
68. Complication of morphin sulphate?
A. Respiratory suppression
B. Hypersensitivity
C. Vomiting
D. Constipation
69. Iron rich food?
A. organ meats
B. green leafy vegetables
C. citrus fruits
D. Canned foods
70. Home care nurse conducting a health programme for screening lifestyle diseases by checking BP
and blood glucose. This type of prevention is called.........................
A. Primary prevention
B. Secondary prevention
C. Tertiary prevention
D. Primodial prevention
71. A patient with severe depression is on anti-depressants after taking the drug the patient shows
restlessness. What would be the reason?
A. As a side effect
B. Normal response
C. Therapeutic response
D. Adverse drug reaction
72. What are the complication of blood transfusion?
A. Transfusion reaction
B. Arrhythmias
C. Hemorrhage
D. Renal caliculi formation

Compiled by : V.Devi | 9176868900 Page 11


Diya Coaching Centre for Nurses
One candle wipes out darkness…….

73. Among the following which is the complication of renal surgery?


A. Renal failure
B. Wound dehiscence
C. Wound infection
D. Hemorrhage
74. List down the steps of nursing process in order
A. Assessment, nursing diagnosis, goal, intervention, implementation, evaluation.
B. Nursing diagnosis, goal, intervention, assessment, implementation, evaluation.
C. Intervention, implementation, evaluation, assessment, nursing diagnosis, goal.
D. Assessment, nursing diagnosis, goal, implementation, intervention, evaluation.
75. List the foods that are rich in vitamin C?
A. fresh fruits and vegetables.
B.organ meats
C. pulses
D. wheat & wheat products.
76. Type of stool present in transverse colon?
A. Semi-solid
B. Solid
C. Liquid
D. Mesh
77. After restoration od ear, what is expected outcome after 5 days?
A. Decreased the hearing
B. Normal hearing
C. Increased hearing
D. None of the above
78. Medical management of parkinson’s disease?
A. Levodopa
B. Methyldopa
C. Antibiotics
D. None of the above
79. What is the antidote of pethedine?
A. Flumazenil
B. Diazepam
C. Promethazine
D. Nalaxone
80. What is the drug of choice for Rheumatic fever?
A. Benzyl pencilline
B. Benzathene pencilline
C. D-Pencillamine
D. Nistatin
81. A Women has pain in the iliac region and feel tenderness on palpation. Which diagnostic
measure the doctor will order now?
A.X-ray
Compiled by : V.Devi | 9176868900 Page 12
Diya Coaching Centre for Nurses
One candle wipes out darkness…….

B. USG
C. CT
D. Blood test
82. A patient after mastectomy should do which all types of exercises after 7 days of surgery?
A. Aerobic exercise
B. hand wall climbling exercise
C. Quadriceps sitting exercise
D. Gluteal setting exercise
83. A patient with mastoidectomy having perorbital edema. The nurse should do which intervention
first?
A. Lower the head end of bed
B. Check BP
C. Head end elevation
D. Foot end elevation
84. A child after surgery for tetrology of fallot at home develops breathing difficulty. Which position
the child should be placed?
A. prone
B. knee chest
C.Supine
D. Sims
85. The patient who is taking spirinolactone the nurse should remain which intervention with the
patient?
A. Limit K intake
B. Take medicine at bed time
C. Limit Na intake
D. Limit calcium intake
86. A patient with 80% burns admitted to hospital. What is the priority nursing diagnosis?
A. Impared body image
B. Self care deficit
C. Fluid Volume deficit
D. Risk for infection
87. A G2P1, mother with pre-normal delivery came to hospital at 20 week of gestation. USG shows
normal gestational age. For the second visit when she came at 28 week, her abdominal height shows
32 week gestation. What would be the reason?
A. Any uterine fibroid
B polyhydraminos
C. Wrong gestational age
D. Oligohydraminosis
88. An 18 month old baby after surgery received in PACU. Baby is spontaneously opening eye, an
independent movements of extremities seen. (what is baby’s GCS)
A. 15
B.10
C. 8
Compiled by : V.Devi | 9176868900 Page 13
Diya Coaching Centre for Nurses
One candle wipes out darkness…….

D. 12
89. Immediately after delivery baby with apgar score is 8 and after 5 minutes the score is 10 which
would be the next measuring prority?
A.O2 administration
B. prevent heat loss
C. Suctioning
D. Prevent infection
90. A patient in found to be unresponsive, at what rate CPR to be started?
A.100
B.>100
C.<100
D. Atleast 100
91. Adverse effect of clozaril (Anti-psychotic)
A. WBC
B. RBC
C. LFT
D. Platelet count
93. Which one is dorsalis pedis artery?
a. A
b.B
c.C
d.D
94. Which one right median nerve?
aA
b.B
c.C
d.D
95. Not risk factor of Angina pectoris?
A. Coronary artery disease
B DM
C. Smoking
D. Decreased total cholesterol level
96. A 10 year old boy C/o fever since last 3 days. Along with dizziness and small rashes all the
body. There are some small red spots with bluish white centre appeared on his buccal mucosa for
last 2 days
A. Mumps
B. Diphtheria
C Measles
D. Chicken pox

Compiled by : V.Devi | 9176868900 Page 14


Diya Coaching Centre for Nurses
One candle wipes out darkness…….

97. 10 year old osteogenic carcinoma boy admitted for chemotherapy and radiation. In which stage
is his cancer
A. I
B.II
C.III
D.IV
98. Tricuspid beat exactly heared on
A. 2nd Rt ICS
B. Apex of heart
C. 4th LTICS
D. 3rd Lt ICS
99. Multiple sclerosis assessment findings except one is;
A Abnormal reflexes
B. Spasticity of lower extrermities
C. Normal bladder function
D. Confusion and euphorea
100. A 10 years old C/o eye irritation on Lt eye during visual examination doctor find that small
scratches on left eye. But right eye is normal. The Prescribe antibiotics eye drops it should install in
A. Left eye in lateral canthus
B. On the left eye with supine portion
C Both eyes
D. On the left eyes with lawless position
101. How can you avoid insulin lipodystropy?
A. Discontinue insulin
B. reduce the dose
C. Rotate the site
D. Massage the site after each injection
102. In which part use can suspect solid fecal material
A.Transverse colon
B. Ascending colon
C. Sigmoid colon
D. Ileum
103. A patient’s ICD is accidently fell down during position change. What’s the nurse’s initial
action?
A. Call the physician
B. Re-insert the tube
C Sterile petroleum soaked guaze dressing on the site
D Keep the tube in bedside bottle.
104. How can a nurse reduce infection in the CVP site?
A. Dressing periodically.
B. Proper hand washing
C. Sterile dressing

Compiled by : V.Devi | 9176868900 Page 15


Diya Coaching Centre for Nurses
One candle wipes out darkness…….

D. Restrict the visitors


105. MRSA precautions?
A. Reverse isolation
B. Contact isolation
C. Droplet isolation
D. Airborne isolation
106. Antidot for calmpose? (diazepam)
A. Flumazenil
B Diazepam
C. promethazine
D. Nalaxone
107. A 91 days boy C/o lethargy and tiredness for last 2 days. He came to the clinic and nurse
noticed that (picture view> tongue >under bluish spot with white center) What signs and
symptoms.(koplicks spot)

A. Neuritis
B Myocarditis
C. Measles
D Rubella
107. 70 year old lady complaints of fatigue and breathing difficulty during walking. Her vitals are
normal. Nurse notify that her tongue is swollen red beefy colour. What is the best advice?
A Iron administration
B. Blood transfusion
C. B12 administration
D. Folic acid
108. A 11 year old girl was playing in the garden suddenly developed rashes on her hand and face.
Mother bought her to the hospital. Nurse noticed that girl’s lips are swollen and tender. What is her
initial action?
A. Epinephrine S/c
B. O2 administration
C. Pain medication
D. IVF
109. What is the following process
Picture

A. Osmosis
B. Diffusion
C. Ultra filtration
D. Hydrostatic pressure
110. A wound picture is given(wound in 5 cm, pink and granulating yellow shield present)
Compiled by : V.Devi | 9176868900 Page 16
Diya Coaching Centre for Nurses
One candle wipes out darkness…….

What type of wound is this?


A. Arterial.
B. Venous
C. Criminal
D. Negligence
112. A patient has 3rd stage of decubitus ulcer. The wound is non exudates and non proliferate what
is the treatment of choice?
A. Hydrocolloidal
B. Hydrogel
C. Adhesive transparent tape
D. Sterile guaze
113. A nurse instructs a community education class on breast health. Which statement indicates
understanding of the appropriate age to starts screening mammogram?
A. At menopause
B. At 65- years old
C At the cessation of breast feeding
D. At 40 years old
114. A patient is taught how to perform a breast self examination by a nurse. Which statement is
best described the understanding of the proper procedure for doing a breast self exam?
A Use the palm of the hand to feel for lumps
B. Apply three different levels of pressure to feels breasts tissue.
C. Stand when performing a breast self exam
D. Perform self exam annually
115. When carrying for a child with ineffective airway clearance related to increased mucus
production the nurse should encourage fluids to
A. Maintain nutrition
B Prevent boredom
C. Stimulate coughing
D. Thin secretions
116. A 59 year old patient with lung cancer and metastases to the bone is in the hospital for pain 10
in number scale(10 point scale) The best goal for this nursing diagnosis is
A. Show no objects signs of pain
B. Not complain of pain
C. State pain is at tolerable level
D. State that all pain is relieved
117. A patient with advanced skin cancer exhibiting cyanosis and edema of head and upper
extremities. Which of the following intervention is most likely provide an immediate benefit for this
patient?
A. Place in a trendelenburg position.
B. Position on the right side
C. Elevate the head of the body
D. Elevate extremeities
118. If a patient developes a complication during blood transfusion, the nurse first should be to do?
Compiled by : V.Devi | 9176868900 Page 17
Diya Coaching Centre for Nurses
One candle wipes out darkness…….

A. Stop the transfusion


B. Notify practitioner
C. Administer antihistamine
D. Administer anti-inflamatory medicines
119. Which of the following types of health care is an example of primary level of care?
A. Diagnosis
B. Acute care
C. Restoration
D. Immunisation
120. In planning for the care of patient with Crohn’s disease the nurse and the patient discuss the
interventions. Which of the following treatment modalities would most likely be considered a
primary intervention for this disease?
A. Surgery
B Medication
C High residue diet
D. Blood replacement
121. A patient with acute Crohn’s disease has been prescribed an elemental diet. The most likely
rationale for this is to
A Reset the bowel
B Improve nutrition
C Improve medication absorption
D. Prepare for surgery
122. A patient has a 6 year history of inflammatory bowel disease that is resistant to medical
therapy. The patient can BEST decreased the likelihood of the disease progressing to
A. Consuming only elemental foods.
B. Stopping smoking
C. Using effective birth control
D. Avoid over-heating
123. A home health nurse is setting up a medication administration schedule for an elderly patient.
The patient is taking oscul (calcium carbonate), Feosul (ferrous sulphate) and orazine (zinc
sulphate). The patient meals at 8 am 12 noon and 6 pm. Which of the following medication
sdministration times would the nurse most likely implement for this patient?
A. Oscal, orazine, feosul at 8 am
B. Oscal at 8 am, orazine at 12 noon and feosul at 6 pm
C. Oscal and feosul at 12 noon and orazine at 6 pm
D. Orazine at 8 am, oscal at 12.00 noon and feosul at 6 pm
124. A community health nurse is administering tuberculine skin tests purified protein derivative,
which of the following time frames should tell the patient to return to the clinic for the test to he
read?
A. In 12-24 hours
B. In 24-36 hours
C. In 36-48 hours

Compiled by : V.Devi | 9176868900 Page 18


Diya Coaching Centre for Nurses
One candle wipes out darkness…….

D. In 48-72 hours
125. A patient who is scheduled for a tonsillectomy is in pre-operative units.The nurse notes an
order for pre-anesthesia medication to be given on call to operation room. The nurse should give
this medication
A. Immediately upon being notified to prepare the patient for transport
B. When the operation room staff arrives to transport the patient
C. Only if clearly needed after assessment.
D Upon the patients arrival in the operation room
126. A patient recently underwent coronary artery bypass graft (CABG) surgery. The nursing
diagnosis includes sleep deprivation related to intensive care environment. The goal for this
diagnosis would be that the patient
A. Gets 4 hours of uninterrupted sleep during the night
B. Takes naps during the day
C. Is free of pain in the first hour post surgery
D. Ambulate 3 hours post surgery
127. The nurse is assisting a patient to ambulate in the hall. The patient has a history of coronary
artery disease(CAD) and had coronary artery bypass graft surgery (CABG) 3days ago. The reports
chest pain rated 3 on a scale of 0 (no pain) to 210 (severe pain). The nurse should first
A. Determine how long it has since the patient last dose of aspirin.
B. Obtain a chair for the patient to sit down
C. Assess the patient’s radial pulse
D. Ask the patient to take several slow deep breaths
128. Which of the following discharge plans take priority for a patient who underwent coronary
artery bypass
A. Instruct on maintenance of low cholesterol
B. Instruct on recognizing signs and symptioms of complication
C. Instruct on the importance of maintaining physical excercise.
D. Instruct on occasion of tobacco use
129. A urse is providing care to the patient having new skin graft on the left leg. The patient is upset
and the nurse notes copious red drainage oozing around dressing. The nurse should immediately?
A Lift the dressing to assess the area.
B. Ask the patient is having any pain
C Apply firm pressure for 10-15 minutes
D. Assess the apical pulse
130. An elderly patient had surgery 2 days for intestinal obstruction vital signs at 10:00 am are temp
37.50 C HR-86, BP-132/72 pain level of 4 of 0 to 10. Abdominal dressing done is dry and intact. The
nasal gastri tube to low intermittent suction. The pt is on strict input and out put every 2 hours.
At 12.20 pm the patient complains abdominal pain, upon assessment the vital signs are T:37.5 C
HR-98, RR-24, BP-146/88 pain level 8 out of 10. The patient abdomen is distended and rigid the
dressing is dry and intact. The nurse should first:-
A. Reposition the pt on Rt side
B. Irrigate the nasal tube to check for patiency
C. Medicate the pt for pain as ordered
Compiled by : V.Devi | 9176868900 Page 19
Diya Coaching Centre for Nurses
One candle wipes out darkness…….

D. Increase the suction on his nasal gastric tube ti high intermittent suction
131. The traction and the urinary catheterhas been dis continuedfor a patient who was immobilized
in traction for weeks. The patient is now having a problem with urin aryincontinence. Which of the
following interventions would the nurse most likely implement?
A. Behavioral training
B. Bladder training
C. Scheduled following
D. Prompted voiding
132. To reduce the risk of treatment methicillin resistant staphylococcus aurevs (MRSA) from an
infectious wound which of the following standard precautions should be implemented?
A. Airbone
B. Contact
C. Droplet
D. Reverse isolation
133. A patient experiencing intermittent claudicating in the legs while at rest which of the following
nursing actions should the nurse takes?
A. Vigorously massage the legs.
B. Place ice on the ankles every 20 minuts
C. Elevate the leg to the heart level
D. Position the leg in the dependent position
134. The nurse is caring for a patient with chest tube connected to closed suction. The nurse should
make sure that which of the following remains readily available at the patient bedside?
A. Sterile towel
B. petroleum gauze pads
C. Normal saline solution
D. Sterile gloves
135. While preparing, preperative paper work for a patient scheduled for neurosurgery, the nurse
asks about the patient’s use of medications. The patient reports taking an aspirin tablet every day.
But has not taken it today. The patient has had nothing by mouth since midnight of the day before,
the nurse should.
A. Inform the enesthesiologist innediately
B. Tell the patient the surgery must be rescheduled
C.Record the information on the form in red ink
D. Obtain a blood sample and notify the attending physician
136. A 35 yrs old female has an inherited gene mutation for achondroplasia an autosomal dominate
genetic disorder. Her husband does not have gene mutation. In planning genetic counselling for this
pt the nurse would be most correct in including which of the following statements regarding the
risk of their children inherited the genetic mutation?
A.Each child has a 50% chance of inheriting gene mutation
B. Female children have 50% chance of inheriting gene mutation
C. Male children will not inherit the gene mutation
D. All female children will inherit the gene mutation

Compiled by : V.Devi | 9176868900 Page 20


Diya Coaching Centre for Nurses
One candle wipes out darkness…….

137. A pt is one day P.OP repair of a large umbilical hernia. The pt complaints of abdominal pain
and described feeling the sutures give way. Upon assessment of the abdomen the nurse observe an
evisceration. The nurse immediate response should be to?
A. Medicate the pt for pain
B. Instruct the pt to cough hard
C. Have patient perform valsalva manuver
D. Cover the abdomen c sterile soaked dressing
138. A 3 yrs old child is seen at the pediatrician’s office. The parents reports the child has had
vomiting and diarrhea per past 15 hours. The child’s is ie thragic with the following vital signs.
Temperature 37.2 0c (99.00f). Heart rate 145, respiration rate 25, and blood pressure level
195/55mmhg. Which of the vital sign is abnormal
A. Temperature 37.2 0c (99.0f)
B. Heart rate 145
C. respiration rate 25
D. Blood pressure level 95/55mmhg
139. A home health nurse is teaching a family member about the care of a patient’s peripherally
inserted central catheter (PICC) which of the following would be appropriate for the nurse to make?
A. Place the use intravenous tubing in a leak proof container and then this seated container
inside a secod leam proof container.
B.You will need to put on a disposable face mask before you connect the intravenous tubing to the
pott of the PICC.
C. The port of the PIC catherter will need to be cleansed with betadine after the infusion is
compleated.
D.The empty medication container can be placed in the same container as your house hold reffuses.
140. While obtaining the pre operative history of a patient schedules for cosmetic surgery, the most
valuable skill at the nurse disposal is
A. knowledge of the procedure
B. Time management skills
C. Listening skill
D. Empathy
141. A community health nurse screens a group of high risk adults for tuberculosis. Which gauge
needle should the nurse use for an intradermal injection on the volar surface of the forearm.
A. 16 gaugue needle
B. 20 gaugue needle
C.22 gaugue needle
D.26 gaugue needle
142. A patient hospitalized with tuberculosis (TB) has a productive cough and hemoptysis. Which of
the following types of isolated room would be the best choice for the patients?
A. reverse isolation.
B.Standard isolation
C. Positive- pressure
D. Nagative pressure

Compiled by : V.Devi | 9176868900 Page 21


Diya Coaching Centre for Nurses
One candle wipes out darkness…….

143. A patient diagnosed with tuberculosis is prescribed isoniazide (isoniazid) rifampin (rifadin)
phyrazinamide (rifamate) ethambutol HLL (Myambuon) and acetaminophen. One month later the
patient with hepatitis which drugs is the most likely cause?
A. Ethambutol (myambuton)
B. Acetaminophen
C. Isoniazid (Isoniazid)
D.pyrazinamide(Rifamete)
144. A patient with tuberculosis can transmit disease to another individual though;
A. Air droplet
B. physical contact
C. Hand to mouth exchange
D. Blood and body fluids
145. A patient recently underwent joint replacement surgery. Which of the following nursing
diagnosis takes PRIORITY?
A. Risk for peripheral neurovascular dysfunction
B. Deficient knowledge on appropriate activity precaution
C. Impaired physical mobility
D. Sexual dysfunction related to pain
146. The parents are anxious after the doctor tells that child needs surgery. The assess parents ability
to cope with this anxiety. Which of the following questions should the nurse ask
A. Dis you know that feeling anxious about your child’s surgery is normal
B. Can you wait until after surgery to begin to cope with being anxious?
C.How do you think feeling of anxiety will affect your child?
D. What has helped you when you felt anxious in the past?
147. 4 years old child brought to the community health clinic for schesuled immunization The child
shiuld receive
A. Varicella, rotavirus, pneumococcal and hepatitis B
B. Measles, mumps, rubella and varicella
C. Rotavirus and inactivated poliovirus
D. varicella and haemophillus influenza
148. The nurse is conducted a community based educational program about diabetes mellitus.
Which of the following statements by a participant would indicate correct understanding of the
teaching?
A. lantus insulin can be mixed with other insulin
B. It is necessary to wipe off the top the insulin vial with alcohol to prevent infection
C. Insulin will changed color after opening
D. Needles can be placed in a hard plastic container with a tightly secure lid
149. A child is treated for superficial (first degree) thermal burns to the thigh The child is in great
discomfort and does not eat. Which of the following diagnosis should receive PRIORITY?
A. Altered nutrition
B. Impaired skin integrity
C. Risk for infection
D. Acute pain
Compiled by : V.Devi | 9176868900 Page 22
Diya Coaching Centre for Nurses
One candle wipes out darkness…….

150. The nurse calls together an inter disciplinary team with members from medicine, social service
and nutritional service to care for a patient with a terminal illness. Which of the following type of
care the team most likely is providing
A. Palliative
B. Curative
C.Respite
D.Preventive
151. A nurse makes ahome visit to a patient recently diagnosed with chronic obstructive pulmonary
disease (COPD) which of the following should the nurse teach the patients about managing COPD?
A. Recognizing signs of impending respiratory infection
B. Limiting fluids intake minimize bronchial secretion
C. Correct technique to auscultate the lung fields
D. Importance of starting antibiotic therapy
152. A patient with chronic obstructive pulmonary disease (COPD) experiencing frequent episodes
of severe dysprea which of the following excercise would teach the patient how to be better control
breathing?
A. Lower side rib
B. Segmental
C. pursed lip
D. Diaphragmatic
153. A patient with copd is experiencing frequent episodes of severe dysphnoea. Which of the
following breathing execises would the nurse most likely implement?
A. Lower side rib
B. Segmental
C. pursed lip
D. Diaphragmatic
155. In evaluating the appropriateness of various excercises enjoyed by a patient with osteoporosis
the nurse would recommend
A. walking
B. Bowling
CSit-ups
D. Golf
156.A patients to the clinic with a “ pins and needles” sensation of the left foot and complaints that
objects appear ‘ Shimmering”. The patients is diagnosed with optic neuritis and reffered for further
testing. The patient is MOST likely to be tested for
A. Glaucoma
B. Multiple sclerosis
C. Lesion of brain stem
D. sychosis
157. A 3 years old has returned to the clinic 4 days after being diagnosed with gastro enteritis and
dehydration A parent reports that the vomiting has stopped and the child is tolerating liquids, rice,
apple sauce and bananas. The diarrhea persists, but seems to be decreasing in volume when
evaluating for signs of dehydration, the nurse will assess the patient’s skin turgor by
Compiled by : V.Devi | 9176868900 Page 23
Diya Coaching Centre for Nurses
One candle wipes out darkness…….

A . grasping the skin over the abdomen with two fingers raising the skin with the two fingers.
B. Grasping the skin over the forehead with two fingers and raising the skin with two fingers
C. Holding the patients mouth open and assessing the tongue for deep creases or furrows.
D. Drawing two tubes of blood and running blood urea nitrogen and creatine.
158. A patient is admitted for pain management due to lung cancer with metastasis to bone with a
nursing diagnosis of alteration in comfort the nurse would anticipate the best short term goal would
be to;
A. Not complaint of pain
B. Appear comfortable and sleep well
C. Verbalise that pain is releaved
D. verbalised that pain is tolerated
159. A nurse is assessing a paetient who just arrived in the emergency department after amotor
vehicle collision. The patient has astrong smell of alcohol on the breath, is restless, and has abluish
discoloration on the abdomen by the umbilicus the patients vital signs T:37:I20 Beats/min
RR:24/min and BP:100/62 mm of hg While other members in the team are evaluating the patient
the nurse should obtain
A. A pair of elastic support stocking
B. A chest tube insertion tray
C. Supplies for peritoneal alvage
D. A vial of Hydralazinc
160. While caring for a terminally ill preschool aged child whose death is not eminent, the child asks
the nurse, am I going to die? The best Nurse response is
A. I’m not sure what is wrong with you, but i hope not
B. Don’t worry when you die you will be angels
C. We all die someday, but you are not going to die today or tomorrow
D. I can’t talk to you about that, you will have to ask your doctor
161. A patient with chronic obstructive pulmonary disease complains of a frequent cough, bilateral
wheezing is auscultated in the lung fields The nurse administers albeterol Neubulizer treatment as
ordered and educates the pt on way to decrease exacerbation. Which of the following action
indicates that the patient understand instruction?
A The patient reduces numbers of cigarettes smoke day
B. The patient requested a pneumococcal vaccination
C The Patient increases sodium and potassium intake
D. The patient exercise whenever experiencing shortness of breath
162. The nurse administers albeturol nebulizer to a child with Asthma exacerbation. The nurse
measures pulse oximetry and auscultates the lungs to determine whether the goal of clear
respiratory status has been met. The step of nursing process is called ?
A. Assessment
B. Diagnosis
C. Implement
D. Evaluation

Compiled by : V.Devi | 9176868900 Page 24


Diya Coaching Centre for Nurses
One candle wipes out darkness…….

163. The home care nurse observes that the asthmatic patient has a cough wheezing. The nurse
administers an albuterol (provenul) nebulizer treatment as ordered. Which type of implementation
is this?
A. Discharge planning
B. Instruct
C. Monitoring and surveillance
D. Therapeutic interventions
164. A child with asthma has an order for albuterol prior to administration of the medication the
nurse must:
A.Pre-oxygenate the patient
B. Obtain venous access
C. Assess the patient’s heart rate
D. Feed the patient a snack
165. When administering albuterol to a child with asthma the nurse should bserve for signs of what
major side effct t this mediction?
A. tachycardia
B. renal failure
C. pneon
D. blurred vision
166. child with sthm is experiencing hick respiratory secretions resulting in increased wrk f
brething the best nursing intervention is to;
A. Encourge fluids
B. Eliminate dairy products
C. Decrease reltive humidity f the rm
D. make the child ly n the left side
E. Eliminate fluids
168. What would be he long term golf r child with asthma?
A. Quickly reverse ir flw obstruction
B. correct hypoxemia
C. deliver humidified oxygen vi nasal cannula
D. Develop a home and school mngement plan
169. The nurse administers an albuterol nebulizer on a child with asthma exacerbation. When of the
following indicates effectiveness of treatment?
A. dventitious breth sound with cough
B. O saturation
C. Nasal flaring
D. respiration rte 28
170. A child is dignosed with asthma exacerbation which of the following nursing diagnoses should
be the first priority?
A. ineffective airway clearance related to bronchospasm and mucosal edema
B. Fatigue related to hypoxia
C. Anxiety related to illness and loss of control
D. Deficient knowledge related to potential side effect of the medication
Compiled by : V.Devi | 9176868900 Page 25
Diya Coaching Centre for Nurses
One candle wipes out darkness…….

171. An asthmatic patient presents with wheezing and coughing. Oxygen saturation is 88% on room
air. Which of the following nursing diagnosis would take priority?
A. Imbalanced nutrition related to decreased food intake
B. Activity intolerance related to inefficient breathing
C. Anxiety related dyspnea and concern of illness.
D. Ineffective gas exchange related to bronchospasam
172. The nurse is visiting the asthmatic patient at home to reinforce the importance of eliminating
environmental allergens and to assess the patient’s response to the environment al chnges. The type
of implementation is cancelled
A. Supervision and coordination
B. Discharge plnning
C. Monotoring and surveillance
D. Therapeutic interventions
173. The parent of a child with chronic asthma is hesistant to discipline becuse the child doesn’t feel
well. The nurse should encourage the pt to
A. set consistent behaviour limits
B. Be more lenent during time of illness
C. Cherish the limited time the child has to live
D. void upsetting the child with limit setting
174. In developing the care plan for hospitalized 3 years old child with asthma the nurse plan to talk
calmly in an appropriate language and explains all procedures. Which of the following statement by
the best demonstrates implementation of the approach?
A.You can use stethoscope to listen to your heart rate and your dolls and then I’ll listen
B. You must not wiggle while I listen to your heart you can hold your doll
C. The stethoscope will feel cold on your chest you can tell your doll how cold it is
D. Let’s go for your doll and place your hands on your tummy while I use stheth.
175. A nursing process which involves the performance of a nursing plan care is called
A. Assessment
B. Nursing diagnosis
C. implementtion
D.Evaluation
176. A ptient who is receiving chemptherpy hs a platelet count of 49,000mm cube(norml value
1,50000 to 4000,00) which of the following nursing action is necessary
A. Minimize invasive procedure
B. Crush oral mediation
C. limit intake of vit.k rich foods
D. monitor the temperature every 4 hourly
177. An elderly patient with long history of diabetic mellitus comes in for a routine checkups. Which
of the following nursing diagnosis would be the nurse anticipates?
A. Risk for impaired skin integrity related to decrease sensation and circulation
B. Excess fluid volume related to disease process
C. Risk for injury to decrease gastric mobility and stress response
D. Deficient fluid volume related to diarrhea and loss of fluids and electrolytes?
Compiled by : V.Devi | 9176868900 Page 26
Diya Coaching Centre for Nurses
One candle wipes out darkness…….

178. A 3 yrs child is brought to the office by the prents who have been toilet trinig the child for the
past 5 months with little success. The parent has been using rewards for keeping the child clean and
dry. Today the prent relized that the child abdomen was very firm, the appetite was poor, and there
had not been bowel movement for 6 days with nursing diagnosed of alternation in bowel
elimination, what is the best goal?
A.A child will recognize the urge to defecte dily
B. The prent will use prise when the child defets in the toilet
C. Predicable, regular bowel hbits will be restored and maintained
D. Toilet training will be delayed. Until the child is congnively ready
179. The nurse is teaching a patient bout spironolactone (aldactone)... which of the following
instructions should the nurse review with the patient?
A. Increasing intake of foods that are high in potassium
B. Taking the medication right before going to sleep
C. Avoiding seasoning that are labeled as salt substances
D. Scheduling the medication so that multi vitamin is taken an hour later
180. Which of the following best indicate effective treatment of meningitis with an intravenous anti-
microbial agent?
A.Severe hedche
B. Negtive kerning sign
C.Nuccal rigidity
D. Photophobia
181. A 32 year old female comes in for evaluation 14 days after an uncomplicated cesrean section.
The patient’s very anxious and complining of shrp stbling pain in her chest. The ptient has dyspnea,
tachypnea and hypoxemia. Which of the following post operative complication is likely?
A. Pulmonary embolism
B.telectsis
C.Pneumonia
D. Aspiration
182. home care nurse reviews the laboratory results for a post partum patient who had acaesarian
section. Which of the following indicates possible wound infection?
A. Incresed WBC
B. Decreased heamtocrit
C. Increased haemoglobin
D. Aspiration
183.Three days ago patient underwent an invasive surgery with an open wound. The patient is
febrile with drop in blood pressure. Laboratory test result shows elevated while blood cell count.
This could be possible
A. Sepsis
B. Atelectsis
C. Internal haemorrhaging
D. Excess fluid volume

Compiled by : V.Devi | 9176868900 Page 27


Diya Coaching Centre for Nurses
One candle wipes out darkness…….

184. A conscious victim of motor vehicle accident arrives at the emergency department. The patient
gasping for air, is extremely anxious, and has a deviated trachea, what diagnosis should the nurse
anticipate?
A.Pleaural effusion
B. Tension pneumothorx
C. Pneumothorax
D. Hemothorax
185. ptient brought to the emergency room with severe head injury. A craniotomy is performed to
evaluate ablood clot. Which of the following is desired expected out comes 24 hour post
operatively?
A. Gg reflex present
B. cerebrl perfusion pressure, 68 mmhg
C. Intracrnial pressure, 2 Immhg
D. Decreased lacrimation
186. A nurse is assigned to patient who is scheduled for n above knee mputation of the left leg.
During the pre-operative preparation, the nurse should ask the patient to?
A. Write yes on the leg
B. Write on the right leg
C. Draw on arrow on the left knee pointing upward
D. Draw an arrow on the left knew pointing downward
187. A patient who is 18 hours post operative after an above the knee amputation complain of
feeling like something is crawling under the dressing as well as increase pressure of the dressing.
The nurse suspects hemorrhge. The patients vital sign remins within the normal range . What
should the nurse do first?
A. Call the physician
B. Plce ice round the dressing
C. Encourage patient to discus fears
D. Lower the temperature of the room
188. patient finds their 2 weeks old infant unresponsive. Infant is limp, Cyanotic and pale there is
no respiration while the skin is cold to touch. The parent begin resuscitation and is the infant
transferred to the hospital where the infant expires. The most likely diagnosis is
A. Sudden infant death syndrome
B. Apparent life-threatening event
C. Apnea of infancy
D. Apnea of unknown origin
189. A neonatal nurse performs Apgr assessment at 1 minute of breath to evaluate the physical
condition of the newborn and immediate need for resuscitation at 1 minute. Apgar scores 7, at 5
minutes Apgar score is 10, the progression of scores suggests.
A. healthy newborn
B. The need for supplement oxygen
C. genetic defect
D. The infant is becoming stable

Compiled by : V.Devi | 9176868900 Page 28


Diya Coaching Centre for Nurses
One candle wipes out darkness…….

190. The nurse is caring for full term newborn who was delivered vaginally 5 minutes ago. The
infant’s APGR score was 8 at one minute at 1 at 5 minute. Which of the following has the highest
priority?
A. Maintaining the infant in the supine position
B. Assessing the infant’s red reflec
C. Preventing het loss from the infant
D. Administering humidified oxygen to infant
191. which of the following can be use determine if a prescribed pain management therapy is
effective for anon verbal patient?
A. rest well
B. taking a sleep
C. talking too much
D. feeling good
192. While caring for aneonate with a meningocole the nurse shuld avoid positioning the child on
the
A. Abdomen
B. Left side
C. Right Side
D. Back
193. A patient with exacerbation of CHF has a nursing diagnosis of excessive fluid volume. The
nurse monitors fluid intake and output and administered furesemide s ordered .Which of the
following indicates the efficiency of the intervention.
A. the patient has pitting edema
B. the patient has shortness of breath
C. the patient has a decrease in weight
D. the patient has jugular vein detention
194. A 62 years old has treated for patient congestive heart failure and a nursing. Diagnosis of fluid
volume excess. After diuretic therapy and dietary interventions, the patient has met all short term
goals. The nurse should
A. Revise the care plan with diagnosis of risk for alteration in fluid balance
B. add a new diagnosed of risk of fluid volume deficit
C. discontinue the care plan as the diagnosis is resolved
D. Continue the care plan as written
195. A patient with congestive filure nd sever peripherl edem has nursir diagnosis of fluid volume
excess. What re the two most important interventions for nurse to initiate?
A. Diuretic therapy and intake and output
B. Nutritional education and low sodium diet
C. Dily weights of intake output
D. Low sodium diet and elevate legs when in bed
196. patient has exacerbation of CHF which one of the nursing diagnosis beir excess fluid (lasix).
Which of the following indicates the efficacy of the nursir intervention.
A. The patient has leg edema
B. The patient has shortness of breath
Compiled by : V.Devi | 9176868900 Page 29
Diya Coaching Centre for Nurses
One candle wipes out darkness…….

C. The patient as decreased in weight


D. The patient has jugular vein destention
197. When caring for a ptient with ostomy, the nurse knows that extra ask protection for
peristomal skin is most important for those with a (n);
. scending colostomy
B. Trnsverse colostomy
C. Ileostomy
D. Sigmoid colostomy
198. While evaluating the nutritional intake of a bedridden patient with multiple pressure sores, the
nurse should make sure the patient increase the tke of
A. Protein rich foods
B. Water
C. Food rich in vitamin A
D. fiber rich foods
199. In what position should dyspnoeic patient be placed?
A.Prone
B.Recumbent
C.Semi-fowler’s
D. Trendelenburg
200. A patient presents to the clinic for routine visit and has the following vital sign temperature 37
0 C (98.6 0f) HR82 respiration rate 18, and BP Bo/94mmhg which vil sign is abnormal?
A. Temperature
B. Pulse
C. Respiration
D. Blood pressure
201. femle patient dmitted for bdominl pin complains of generalized paiNuse, Vomitting and
constipation, nursing assessment fins; temp 38.6 0 c (101.5 0 f) heat rte 92, RR 18, BP 130/68 mmhg.
The patient has rebound tenderness and bdomin rigidity in the pst hour. Her pain has localized on
the RT side., The nurse suspects,
A. Intestinal obstruction
B. Influenz
C. Appendicitis
D. Pyloric stenosis
202. community health nurse is implementing an dult immunization program the neighbourhood,
which of the following would most likely be universl recommended sdult vaccination and does
frequency general population
A. Tetanus-diphtheria toxoid every 20 years
B. Pneumococcal vaccination every 2 years
C. Influenza vaccination every year
D. One time typhoid vaccine followed by boosters every 5 years
203. A 6 old patient has been diagnosed with cute rheumatic fever. Antibiotic for this illness is?
A. Bezthgine pencillin (megacillin)
B. Amozicillin
Compiled by : V.Devi | 9176868900 Page 30
Diya Coaching Centre for Nurses
One candle wipes out darkness…….

C. Erythromycin
D. Vancomycin
204. A child is admitted to the hospital with congenital heart disease. Which of the following
nursing diagnosis should receive priority?
A. Decreased cardiac output related to decreased myocardial function
B. activity intolerance related to carbexi
C. Impaired gas exchange related to altered pulmonary blood flow.
D. Imbalanced nutrition less than body requirements related to excessive energy demands.
205. Prior to initiating therapy with infractionted heparin for a patient hospitalized with deep vein
thrombosis the nurse should plan to
A. Weight the patient B
B. dminister spirin
C. Limit fluid intke D
D. Undress the patient
206. Prior to initiating therapy with unfractionated heparin for a patient hospitalized with a deep
vein thrombosis this treatment requires.
A. Bed rest
B. spirin therpy
C. Fluid restrictions
D. A high protein diet
207. A patient with chronic liver disease secondary to patients C has been admitted with a nursing
diagnosis of alternation in nutrition. Less than body requirement the BEST long term goal is the
patient will.
A. gin at least 10% of body weight
B. Attiain nd mintin ideal weight
C. Verbalize understanding nutritional needs.
D. Include high quality protein in diet.
208. The nurse is Assessing a patient with a history of seizure disorder. While checking the patient
vital sign the patient develop rhythmic, jerking movements of the arms and legs. The nurse should
immediately place the patient in which of the following positions?
A. Prone
B. Supine
C. Semi-Fowler’s
D. Lateral
209. nurse is ssessing to care for a child aseizure disorder the nurse observe the child becomes stiff
and lose consciousness. Following by jerking movements for 1minute after which the child becomes
very sleepy. Which of the following types of seizures occurred?
A. Absence (petit mal)
B. Generalized (tonic conic)
C. partial psychomotor (temporal lobe)
D. Status epilepticus
210. Patient is scheduled for an abdominal neurysm repair. This is what type is what type of surgical
intervention?
Compiled by : V.Devi | 9176868900 Page 31
Diya Coaching Centre for Nurses
One candle wipes out darkness…….

A. Diagnostic
B. Transplant
C. Curative
D. Palliative
211.A community health nurse is teaching heath. Class about infectious disease. Process the nurse
class that Rabies would be considered which of the following types of infection?
A. Viral
B. Protozoan
C. Fungal
D. Bacterial
212. A patient receiving chemotherapy developed some raised red.Edematous wheel on the skin.
Which of the following care plan alternatives would most ikely need to occur before the next
treatment?
A. Reinforced relaxation techniques
B. Cntinue chemotherapy without change
C. Continue with radiation therapyonly
D. Premiditate the patient with an antihistamine
213. A 6year old patient has presented to the cinic with fever, malaise and anorexia, the patient was
treated 2 weeks ago for streptococa infection of the throat. The nurse should expect the physician to
order what test?
A. Electro cardiogram
B. Jones test
C. Spinal tap
D. Heart biopsy
214. A community is experiencing an outbreak of staphylococal infections. The nurse instructs
residents that the most common mode of transmission is by:
A. Respiratory droplets
B. Contaminated foods
C. Hands
D. Soil
215. A hospitalized patient has fallen from bed, the nurse notes shortening of the left leg. Pain upon
movements of the left leg and rapid swallow respirations what action should the nurse take First?
A. Call for help
B. Immobilize the left leg
C. Obtain blood pressure
D. Evaluate lung sounds.
216. A community health nurse visit patient who was suffered a stroke. The patient spore explain to
the nurse that the patient chokes on foods at times which of the following referral ordered would
the nurse anticipate needing for this patient?
A. Speech therapist
B. Dietician
C. Physician therapist
D. Neurologist
Compiled by : V.Devi | 9176868900 Page 32
Diya Coaching Centre for Nurses
One candle wipes out darkness…….

217. A 59 year old patient arrives in the emergency department diaphoretic and complains of chest
pain and shortness of breath the patient is sibling states that this has happened before and it is just
anxiety. Upon evaluation the physician diagnosis result of this drug therapy?
A. Balanced between oxygen supply and demand
B. Increase in blood flow to the heart
C. Reduction in oxygen demand and consumption
D. Vessel relaxation
218. A patient with end-stage cardiomyopothy and angina pectoris to the office complaining of
frequent chest pain and severe dyspnea with a nursing diagnosed of alteration in comfort. What is
the BEST short term goal for the patient?
A. Perform all activities of daily living without complains of chest pain or shortness of breath
B. Verbalize and employs strategies to decrease pain and increase coronary blood flow
C. Take pain medications around the clock and use supplement oxygen at all times.
D. Understand the disease process and accept the imitation that it place on his life style
219. A patient has an order for a pneumatic compression device. Which of the following is an
appropriate goal?
A. Reduce the risk deep vein thrombosis
B. Reduce lower extremity edema
C. Reduce lower extremity pain
D. Reduce the risk of phlebitis
220. A patient with severe diverticulitis had surgery for placement of colostomy. The patient upset
crying and will not look at the colostomy. Which of the following would be the HIGHEST priority
Nursing diagnosis at this time?
A. Knowledge deficit, colostomy care
B. Distorted body image
C. Self case deficit, toileting
D. Alteration in comfort
221. A patient came to the emergency department with complaints, dizziness and confusion; clinical
symptoms include tachypnea and dyspnea with the use of accessory muscles to facilitate breathing.
Which of the following orders should the nurse most likely implement to reduce the patients
confusion and disorientation?
A. Oxygen therapy
B. Chest physic therapy
C. Bronchodilator
D. Hydration fluids
222.A 6-month old boy is admitted with a diagnosis of failure to thrive. According to the growth
chart at month of age the infnt’s weight is in which percentile?
A. 25 th
B. 5 th
C. 10 th
D. Below the 5 th

Compiled by : V.Devi | 9176868900 Page 33


Diya Coaching Centre for Nurses
One candle wipes out darkness…….

223. A patient is 2 days postoperative hernia repair and has an order for a dressing change, patient
has been diagnosed with immune deficiency disease syndrome (AIDS). While performing the
dressing change the nurse shoud take which of the following actions?
A. Put the patient in a private room.
B. Wear gloves during the dressing change
C. Wear gloves, gown, mask during the dressing change
D. Put the patient in reverse isolation
224. When administering an enema to adult patient. How far should the nurse insert the tubing into
the rectum?
A. 2.2 to 4.4 cm (1 to 2 inches)
B. 4.4 to 6.6 cm (1 to 3 inches)
C. 6.6 to 8.8 cm (3 to 47 inches)
D. 8.8 to 10.1 cm (4 to 5 inches)
225. A nurse is implementing Nursing interventions to monitor a patient following Kidney surgery.
Which of the following complications would be the most likely post operative risk after renal
surgery?
A. Deep vein thrombosis
B. Hemorrhage
C. Nausea
D. Hemiparesis
226. As a part of Neurological assessment which of the following associated with the highest score
on the Glasgow coma scale?
A. Eye opening to pain no verbalisation
B. Confused obey commands
C. Eye opening to speech confused
D. Localized pain abnormal extension
227. While caring for a patient prior to surgery to amputate the leg. What is the most effective
measure to prevent phantom limb sensation after the amputations?
A. Cntrol pain prior to the surgery
B. Make sure the patient understands the preocedure
C. Elevate the imb on two pillows
D. Help the patient grieve for the limb
228. If a patient develops a complication during a blood transfusion the nurse first action should do
to
A. Stop the transfusion
B. Notify the practitioner
C. Birth control pills
D. Help the patient grieve for the limb
229. A patient n elevated prothrombin (PT) time which medication should the nurse consider as a
possible cause of the elevated PT time?
A. Rifampin
B. Vitamin K
C. Birth control pills
Compiled by : V.Devi | 9176868900 Page 34
Diya Coaching Centre for Nurses
One candle wipes out darkness…….

D. Phenytoil
230. A home care nurse visit a patient with a new ‘below the knee amputation’. The cite of the
incision is red, warm and tender with purulent yellow drainage. The pt has a new prescription for
cephalexin (keflex) nd oxycodone(oxycontin). What would the nurse instruct to do FIRST ?
A. Take oxycodone as soon s possible
B. Take cephalexin as soon as possible
C. Wash the incision site and apply bacitracin cream
D. Wash the incision site and apply hydrocortisone
231. A patient has the following order cephalexin (keflex) 500mg. By mouth 4 times day, the
pharmacy has the following dose 250 mg. Per 5ml. The nurse should administer
A. 5 ml
B. 10 ml
C. 15 ml
D. 20ml
232. A marathone runner experiences a sudden onset of sharp pain in the calf immediately after a
workout. Which of the following would the nurse suspect the patient is experiencing?
A. Bursitis
B. Tendonitis
C. Plantar facitis
D. Joint dislocation
233. A mastectomy ptient has develop[ed lymphedema of the eft arm. The Nurse shoud teach the
patient that hte bst position for the arm is
A. Immobilised across the chest
B. Dependent
C. Elevated
D. In traction
234. A patient in the emergency room 20 cm laceration with the right forearm. The nurse prepare for
which type pf anesthesia to be administered before the laceration is repeated by the physician?
A. Intravenous
B. Regional
C. Local
D. General
235. A nurse in a community health clinic is in change of immunisation. When patient visits the
cinic, the nurse knows that immunisation should be reviewed.
A. At the age they are scheduled to be administered
B. One month prior to recommended immunization schedule
C. At every clinic visit
D. At monthly intervals
236. A child was admitted to the hospital three hour ago with a closed head injury. The child
responds appropriately but sluggishly to stimuli and drift in and out of sleep. Which of the
following best describes this patients level of consciousness ?
A. Lethargic
B. Obtunded
Compiled by : V.Devi | 9176868900 Page 35
Diya Coaching Centre for Nurses
One candle wipes out darkness…….

C. Semi comatose(drowsiness)
D. Comatose
237. A healthy patient is in the doctor’s office for a pre-operative visit before a total replacement.
The nurse interviewing the patient charts, the following medications. Aspirin 81mg once a day,
Vitamin E- 260 international un its once a day and unknown amount of a herbal supplement once a
day. Based on the p-atient medications list, which of the following labs would be important
prepaeratively?
A. Prostrate specific ntigen(PSA)
B. Blood glucose
C. Creatine phosphokinase iso enzymes(CPK enzymes)
D. Prothrombin time.
238. A patient with long standing DM (type 1) is scheduled for surgical amputation of 4 gangrenous
toes on the right foot. Which surgical intervention would this be classified as?
A. Paliative
B. Curative
C. Reconstructive
D. Diagnostic
239. The nurse is caring for a patient who just had chest tube inserted due to a spontaneous
pneumothorax. An appropriate goal is that the patient will
A. Be free of pain within 4 hours
B. Report decreased pain
C. Rest quietly
D. Sleep with few movements
240. A patient with the deep vein thrombosis (DUT) is being treated with a ow molecular weight
heparin (LMWH) the patient reports increased pain in the affected extremely. The nurse observes
the affected extremey has increased in size bu 0.2 cm (0.8 inches) during the past 24 hours. Which of
the following actions should the nurse take?
A. Administer the next dose of LMWH before the scheduled time.
B. Apply dry heat to the site.
C. Elevate the extremity
D. Reinforce the importance of ankle circling Exercises.
241. A physician orders a lactated ringer solution to infuse at 125 cc/hr. This is an example of which
type of solution?
A. Hypotonic
B. Hypertonic
C. Isotonic
D. Hyper alimentation
242. Physician order an intravenous fluid of D5NS at 100.cc /1 hr. This is example of which of the
solution ?
A. Hypotonic
B. isotonic
C. hypertonic
D. Hyper alimentations
Compiled by : V.Devi | 9176868900 Page 36
Diya Coaching Centre for Nurses
One candle wipes out darkness…….

243. A patient is in the pre operative area to Lumbar surgery. The patient reports anxiety bout being
intubated and expresses concern about waking up during the surgery. The nurse must discuss the
patient corcern with the
A. Anesthesia Provider
B. Surgeon
C. Scrub nurse
D. Charge nurse
244.The nurse is caring for a patient dignosed with Human Immune Deficiency virus. Which of the
following nursing diagnose take priority?
A. Diarrhea related to medication side effects.
B. Risk for infections related to inadequate immune system.
C. Imbalanced nutrition related to decreased apetite
D. Impaired tissue integrity related to cashexia and malnourishment
245. A nurse access a 3 month old infant. The patient expresses and feeling over whelmed. The
nurse offer information on available parenting support. This level of child abuse prevention is
classified as which of the following?
A. Intervention
B. Primary
C. Secondary
D. Tertiary
246. The nurse is caring for a patient with a coronary thrombosis who is receiving prescribed
streptokinase (Streptase). The patient reports the onset of rash as well as feeling hot while
experienced chills. The nurse should immediately implement the plan of care for ?
A. Medication side effects
B. An allergic reaction
C. A pulmonary embolus
D. Perphera artery occlusion
247. The nurse is teaching a patient who was just diagnosed with narcolepsy. The nurse should
teach the patient that which of the following typically increases the level of fatigue.
A. Taking brief Naps
B. Participating in an exercise programme
C. Eating large meals
D. working in a co environment
248. The physician has prescribed quinidine poyglacturonate (Apo-Quinidine 8.25mg, every 4 hours
for a patient who weighs 50 kg. The drug is available as a 275 mg tablet. The nurse should
administer how many tablets for each doses?
A. 2.5
B. 2
C. 1.5
D. 1
249. The nurse is teaching the parent of a child with celiac disease. Which of the following diets
should be revival with the patient?
A. Guten-free
Compiled by : V.Devi | 9176868900 Page 37
Diya Coaching Centre for Nurses
One candle wipes out darkness…….

B. Dairy free
C. Vegetarian
D. Sodium restricted
250. A patient has peripheral vascular disease The nursing diagnosis is ineffective tissue perfusion
peripheral, which of the following is n appropriate goal ?
A. Patient wil identify three factions to improve peripheral circulation
B. The patient will have palpable peripheral pulses in one week
C. The patient’s feet will be warm to touch
D. The patient will ambulate the length of the hallway
251. On the second day of hospitalisation for verticuloperifoneal shunt revision, a child with spinal
bifida developed lier itching and wheezing. The nurse should determine if the patient has been
exposed to
A. Peanuts
B. Strawberry
C. Eggs
D. Latex
252. A patient with malignant cancer has decided to stop chemotherapy and receive hospice care.
What is the priority nursing diagnosis?
A. Alternation in comfort
B. Hopelessness
C. Powerlessness
D. Non-compliance
253. A nurse accessing a 16 month old child observes bruises scattered over the body that are at
different stage of healing. The chid also has poor and diaper rash. The goal f treatment for this chid
is to:
A. Ensure the physical and emotional safety of the chid
B. Remove the chid from the parent
C. Admonish the parents of the child
D. Ensure that the child stays with the bioogy
254. While visiting a patient who had a left hip replacement surgery one week ago, the patient
complains to the home care nurse of episodic numbers and finding of the lower left extremities.
Assessment of the patient shows that the lower left extremities are slightly cool to touch when
compared to the lower right extremities. There is no swelling or redness on assessment. What would
be the NEXT nursing intervention?
A. Reassure the patient that this is normal after surgery
B. Refer the patient to the surgeon immediately
C. Encourage the patient to decrease activities involving the left hip and extremities.
D. Refer the patient to a physical therapist immediately
255. A nurse is evaluating a patient 5 days after a right tota hip replacement. Which of the following
goals is appropriate for the patient?
A. Maintain hip abduction without dislocation
B. Rest with legs elevate while sitting
C. Tie shoes and put on undergarments without assistive devices
Compiled by : V.Devi | 9176868900 Page 38
Diya Coaching Centre for Nurses
One candle wipes out darkness…….

D. Perform scissors like leg exercise daily


256. Prior to providing care for a hospitalised infant, the nurse must:
A. Check respiration
B. Maintain a patient airway
C. Check vital signs
D. Hand washing
257.When caring for a patient with new sigmoid colostomy. The nurse knows that the stoma nods
that increase in size?
A. One month
B. Two month
C. Six month
D. One year
258. A 7 week old infant boy is admitted with projectile vomiting decreased urine output decreased
bowel movements and weight loss. He has poor turgor and appears hungry. The nurse observes left
to right peristaltic waves after the vomits. The nurse would expect to find which of the following
during the physical assessment?
A. Hepatosplenomegaly
B. A palpable pyloric mass
C. Lymphadenopathy
D. Bulging fontanelles
259. A nurse will need to change the dressing on a patient’s central venus catleter during the shift.
The nurse would plan to
A. Limit the patient’s activity for an hour dressing change
B. Position the patient on the left side before removing the old dressing
C. Put on sterile gloves after explaining the procedure to the patient
D. Cleanse the insertion site using a circular motion
260. During the operative period, a nurse is assigned to care for a morbidly obese patient with an
abdominal incision. The nurse knows that this patient’s weight increases the risk of
A. Left sided heart failure
B. Pressure sores of the coccyx
C. Conspiration and ileus
D. Wound dehisecence
261. Which of the following takes place during the implementation phase of the nursing process?
A. Development of a goals and a nursing care plan
B. Identification of actual or potential health problem
C. Actualization of the care plan through nursing interventions.
D Deformation of the patient’s responses to the nursing interventions
262. For a patient with a colostomy which of the following interventions is appropriate for
preventing the risk of the impared skin related to exposure expressions?
A. Empty pouch when it is completely full
B. Remove the skin barrier inspect the skin monthly
C. Recaps skin barrier opening to size of stoma with each change
D. Cut an opening in the skin barrier then the circumference of the stoma
Compiled by : V.Devi | 9176868900 Page 39
Diya Coaching Centre for Nurses
One candle wipes out darkness…….

263. An infant who weigh 9kg (19.8165 require 900m of fluid per day for maintenance fluids. The
infant typically consumes 120 ml during each feeding. The infant must have how many feedings per
day to meet the fluid maintenance needs?
A. 4
B. 8
C. 10
D. 12
264. A patient has a pacemaker implanted. Which of the following interventions is appropriate for
the nursing diagnosis for risk for injury?
A. Have pt avoid exposure to MRI
B. Observe incision for redness, warmth, purulent discharge and soreness
C. Offer backrubs to promote relaxation
D. Instruct pt in dorsiflexion exercises of ankles
265. A. Pt undergoing treatment for cancer with bone metastasis is experiencing severe pain. Which
of the following treatment the nurse most likely expect to improve patience pain control?
A. Adjuvant radiation therapy
B. Palliative radiation therapy
C. Curative radiation therapy
D. Radio surgery
266. A home care nurse visits a pt with diabetes. The pt ate 3 well balanced meals sweet desert and
exercises 30 minutes a day twice a week. Also the pt takes hypoglycaemia medication and the blood
glucose level ranges from 150-200mg/dl. The nurse set a goal of eliminating sweet dessert and
increasing the frequency of exercises to three times a week. This week the patient exercised for three
times a week for 30 minutes and ate dessert only after dinner. The glucose range from 100-150
mg/dl. The nurse evaluate that :
A. The goal will not be met
B. Progression is being made towards the goal.
C. The goal is met
D. The goal is inappropriate
267. A nurse is assigned to care for a patient with an ileostomy. The nurse would expect the ostomy
discharge to be
A. Fluid mushy
B. Mushy
C. Liquid
D. Solid
268. A nurse educates a patient diagnosed with diabetes, on the importance of exercise and a well
balanced, low carbohydrate diet. The patient takes metforin (glucophage) 500 mg once a day. which
of the following indicates the patient’s plan of care needs to be re evaluated?
A. Blood glucose level is 90mg/dl
B. Hbg Aic (glychemogobin) level is 9.0%
C. Total cholesterol level is0mg/dl
D. Low density lipoprotein is 130mg/dl

Compiled by : V.Devi | 9176868900 Page 40


Diya Coaching Centre for Nurses
One candle wipes out darkness…….

269. A nurse schedules a patient for surgical procedure to take place in 1 week. When would the
nurse Most likely implement surgical education?
A. After admission to the hospital
B. Start during the visit
C. Immediately prior to anesthesia
D. After the operation
270. The nurse is inserting a nasogastric (NG) tube into a patient as prescribed. The nurse has
advanced the tube into patient’s posterior pharynx. The nurse should ask the patient to
A. Hold the breath
B. Stair upwards will eyes towards the ceiling
C. Perform the valsalive mameuver
D. Lower the chin towards like chest
271. A home care nurse staff a diabetic patient who was started on insuin injection upon
examination, the nurse observe small limps and dents on the right upper where the patient has
injectd insulin, what is the best nursing intervention ?
A. Refer patient to dermatogist
B. Refer the patient to endocrinologist
C. With hold insulin
D. No need of management
272. After cardiac surgery a pt have been prescribed low sodium, low cholesterol. Which of the
following menu is the BEST ?
A. Salami, dry bread
B. Baked chicken thigh sliced tomatoes, ice berg lettuce
C. Pasta with canned tomato sauce, wheat bread
D. Bacon, lettuce and tomato sandwitch with mayonnaise dressing
273. A home helth nurse visit a patient with COPD using home oxygen 21/mt. The reports period of
shortness of breath and enquires about increasing the oxygen to 41/mt. The nurse explains that the
increasing supplemental oxygen wil
A. Increase activity tolerance
B. Suppress the hypoxic drive
C. Activate the shortness of breath
D. Prevent lung infection
274. The nurse should avoid the use of the dorsoglateal site for an intramuscular injection in
children because of the risk for injury to which of the following nerves?
A. Vagus
B. sciatic
C. Ilionguinal
D. Lumbar plexus
275. Twelve hour after removal of a benign liver tumer, the nurse observed that the patient has
decreasing blood pressure, decreasing pulse pressure increasing het and increasing respiratory rate.
The patients skin is cool and pale after lowering head of the bed, what should the nurse do next?
A. Cal the physician
B. Administer pain medication
Compiled by : V.Devi | 9176868900 Page 41
Diya Coaching Centre for Nurses
One candle wipes out darkness…….

B. Position the patient on the left side


D. Apply cool, wet clothes under the arm
276. The nurse is assigned to care for an elderly ptient with ow exudates pressure ales which if the
following types of dressing woud the nurse most likely to use?
A. Hydrogel
B. Hydrocolloid
C. Polyurethane
D. Polyurethane foam
277. A patient with an unsteady gait and history of falls has a care plan inte that including
keeping the walker in research and pathway free of obstacle evaluation after 1 week. The patient has
had no falls but the gait remains un control. The nurse should
A. Continue the plan of care as written
B. Allow the patient to replace the walker with care
C. Allow the patient to ambulate short distance without the walker
D. Have the patient practice stopping over small objects
278. The nurse is caring for a patient who had a totl colectomy 24 hours ago due to a malignant
neoplasm in the rectum. The patient continues to receive intravenous fluid and has started a clear
liquid diet. The nurse understands that the patient is at INCREASED risk for which of the following
postoperative complication?
A. Disseminated intravascular coagalopathy
B. Atelectasis
C. Syndrome of inappropriate anti diuretic hormone(SIADH)
D. Hypokalemia
279. When doing community based teaching for latex allergies, the nurse should plan to teach the
patient that;
A. Food handled by people wearing latex gloves stimulate an allergies responses
B. Foods containing nuts may trigger n allergic cross response in people with latex allergies
C. The patient should wear a face mask while in the hospital due to large amount of airborne latex
D. Houses used on gas pumps contain latex should be avoided
280. The nurse is assessing a 16 month old girl. The nurse observes poor hygiene, diaper rash and
boulses over the child’s body that is at different stages of healing. Which of the following
interventions should reduce fear and promotes the trust of the chid?
A. Avoid scaring the chid by saying “no, or setting limites
B. Challenge the information the parents give regarding the injury
C. Question the parents of the child over the course of hospital stay
D. Assign one nurse to care for the child over the course of hospital stay
281. A patient who is prepared for hip surgery has an order for external pneumatic compression
devices. The teaches the patient that pneumatic compression can help prevent:
A. Upper respiratory infections
B. Decrease breath sounds
C. Deep vein thrombosis
D. Bleeding at the surgical sites

Compiled by : V.Devi | 9176868900 Page 42


Diya Coaching Centre for Nurses
One candle wipes out darkness…….

282. A patient presents with a productive cough with a moderate amount of white forthy sputum
and dyspnea. The patient is anxious and the nurse notices on assessment that the patient is using
accessory muscle including intercostals spaces to breath and has jagular vein distension. The patient
has a history of hypertension nd heart failure. What should the nurse administer first?
A. Digoxin (lanoxin) to improve the ability of the heart to ump effectively
B. Oxygen therapy to combat hypoxemia
C. Furosemide (lasix) to reduce blood volume and pulmonary congestion
D. Morphine sulphate (duramorph) to reduce anxiety
283. A nurse is preparing to meet with an individual whose spouse recently diagnosed with
alzheimer’s disease. The nurse should know that the primary goals of treatment are:
A. Curing the alzheimer’s disease
B. Maximising the functional ability and improve quality of life
C. Having the Alzheimer”s patient placed in a safe controlled environment
D. Making al decision for the patient and conforming to home
284. A community nurse interviews an 87 year old patient diagnosed with Alzheimers disease.
Because the patient provides conflicts information the nurse compares subjective and objective date
to find a possible reason for the conflict data. The process of assessment is called:
A. Data verification
B. Analitical interpretation
C. Mental assessment
D. Subjective observation
285. The nurse assess an elderly patient for health problem. The family reports the patient has
trouble remembering and they are concerned bout Alzheier’s. Why of the following are risk factors
for Alzeimer’s disease?
A. Genetic history and male gender
B. Ethnic group and dietary habits
C. Genetic history and female gender
D. Dietary habits and male gender
286. A pt c Alzhemires disease has a fall which results in # of right hip following the surgical repair
of the fracture the pt is discharge home c family care givers. During home visit to provide wound
care the family verbalizes frustration that the pt has been Incontinent since returning home. Which
of the following instructions would nurse gives to the family?
A. Instruct the family in providing incontinent care and put nsg nursing diagnosis for risk for
impaired skin integrity
B. Suggest to the family that if the stress is overwhelming, placement in a skill nursing facility may
be needed
C. Suggest toileting the pt on a regular schedule and applying incontinence pt at all times
D. assess for the cause of incontinence, and appropriate nsg diagnosis and intervention
287. A nurse assists apt c Alzheimer’s disese in tooth brushing. The pt indicates wanting to complete
the task alone, but is a unable to get the tooth paste on the toothbrush. The nurse can most
effectively help the pt by?
A. Providing privacy to complete the task
B. Completing task
Compiled by : V.Devi | 9176868900 Page 43
Diya Coaching Centre for Nurses
One candle wipes out darkness…….

C. Providing hand over hand assistance c the task


D. Telling the pt to forego brushing the teeth today
288. A child c iron deficiency complains of feeling tired all the time. The nursing diagnosis of fatigue
is related to?
A. A decreased ability of the blood to transport O2 to the tissue
B. An increased paroxysmal abdominal pain and distention to the stomach
C. A decreased anxiety level during hospitalization
D. decreased nutritional intake c malabsorption of nutrition
289. A patient arrives at the emergency room with burns over upper torso arm the nurse should
obtain the patient’s pulse at wich of the following arterial location
. Redial
b. Crotid
C. Femoral
D. Apica
290. A patient with a spinal cord injury states “ I heve no control over my situation, I can’t do
anything for myself”. This patient is exhibiting
A. powerlessness
B. Delusions
C. Suicidal ideation
D. Designation
291. nurse is teaching a prenatal class to a group of the first time mothers, each as different points in
their gestation, which of the statement if TRUE regarding the management of fatigue?
A. Rest flt on back, especially during the third trimester
B. Exercise programs should focus on their training
C. frequent 15 minutes to 30 minuets rest periods are important
D. Six hours of sleep anight is adequate
292. A nurse is caring for a postoperative patient who is an subcutenious, low dose heparin. The
mediation is used to prevent
A. Deep vein thrombosis
B. Congestive heart failure
C. Paralytic ilicus
D. Pneumonia
293. The pt is recovering following surgery for placement of colostomy. The nurse goes to the pt and
instruct how to care for the colostomy. The pt’s roommates has visitors and pt does not want to
participate at this time. What should the nurse do?
A. Document the pt’s refused and non-complience on the care plan
B. Tell the pt that this is vital information and may delay discharge
C. Paln atime convenient to both the pt and nurse
D. Pull the cartain around the bed and speak ensuring privacy
294. Which of the foowing actions would be appropriate for the nurse to take when caring for a pt
on contact precaustions?
A. Serve he pt’s meal on disposable plastic eating utensils
B. Instruct visitors to tak to the nurse before entering the pt’s room
Compiled by : V.Devi | 9176868900 Page 44
Diya Coaching Centre for Nurses
One candle wipes out darkness…….

C. Rinse both hands c water after removing gloves


D. Place a surgical mask on the pt during transport
295. A pt recently diagnosed c herpes zoster. The nurse establishing the care plan would most likely
assign the highest priority to which of the nursing sis?
A. Anxiety
B. Social isolation
C. Peripheral neurovascular dysfunction
D. Acute pain related to disease condition
296. A nurse visit a pt at home who doesnot, understand how to take newly prescribed medicines.
The prescription reds 5 ml PO TID p.c. meals. The nurse explains the patient, the correct way of
taking medicine is
A. 1 table spoon by mouth, three times a day before meals
B. 1 teaspoon by mouth, three times a day after meals
C. 1 teaspoon by mouth, three times a day before meals
D. 1 tablespoon by mouth three times a day fter meals
297. In order to reduce the risk of dieseas transmission from a patient with diphtheria which of the
following standard precaution who would be the nurse implemented?
A. Airborne
B. Contact
C. Droplet
D. Ventilator
298. Patient with measles(rubella is an airbone precaution, which of the foowing would be essential
to implement for non immune person entering the room?
A. Gloves
B. Gowns
C. Face shields
D. Masks
299. A ptient sustained multiple musculoskeletal trauma after a motor vehicle collision and is now
too skeletal traction awaiting surgery. The nurse observes that the patient has developed a large
area of flat fix points purple- coloured areas on the thorax. Which of the following action would be
appropriate for the nurse to do?
A. Discontinue the opioid that is being administerd
B. Place on extra blanket on the patient
C.Reese the weights on the patient skeletal traction
D. Administer diphendydrmine (Benadryl Prescribed patient allergi reaction
300. physicin has ordered gavages feeding every 4 hours for a 12 week old infat with failure to
thrive is order a know how far to insert the feeding tube. The nurse should measure to distance from
A. The infant’s month to the sciphoid process of the sernum
B. The tip of the infant nose to the car and then to the umblicals
C. The infant’s mouth to the ear and flexible amblicity
D. The tip of the infant’s nose to the ear and flex to the sciphold process of the sternum

Compiled by : V.Devi | 9176868900 Page 45


Diya Coaching Centre for Nurses
One candle wipes out darkness…….

301. A nurse is assessing an infant diagnosed with failure to thrive. In addition accurate
anthroprometric measurement, complete nutritional history infant feeding bility, and hed to toe
assessment the nurse should assess which of the following
A. Parent to child interaction
B. Number of sibling in the home
C. Current sleep patterns
D. Exposure to second hand smoke
302. n infant who weights 9kg (19.8 lbs requires 90ml fluid per day maintenance fluids the infant
typically consumes 120ml during each feeding. I infant must have how many feedings per day to
meet the fluid maintanence needs?
A. 4
B. 10
C. 8
D. 12
303. A school nurse refers a child who failed the school vision screening for doctor. The child returns
with glasses to be worn at all times.
A. Redness of the eye
B. Episodes of seixers
C. Improved vision with glasses
D. Lazy eye
304. 2 years old child in the emergency de[partment exhibits symptoms of bacte meinightis. Whixh
of the following tests confirm or rule out this diagnosed?
A. Magnetic resonance imaging (MRI)
B. Mgnetic encephalogram
C. Computed tomography scan(CT)
D. Lumbar puncture
305. A patient exhibit clinical manifestation of a pulmonary embolism. Arterial blood gas (ABG)
levels and a chest x-ray are ordered. Which of the following test is used to diagnose this condition?
A. Computer tompgrphy scan(CT)
B. Magnetic resonance imaging (MRI)
C. Pulmonary function test
D. Pulmonary angiography
306. A patient is admitted to the emergency department with a sucking chest wound has diminished
breath sound on auscultation which of the following interventions would the nurse performed
FIRST?
A. Monitor O2 saturation and DG levels
B. Apply petroleum gause to wound
C. Prepare to patient for emergency thoracentesis
D. Position the patient in an upright position
307. A patient has pulmonary embolism. Which of the following nursing diagnosis has PRORITY
A. Anxiety related to pain, dyspnea and concern of illness
B. Risk for injury related to altered hemodynamic status
C. Acute pain related to congestion and possible lung interaction
Compiled by : V.Devi | 9176868900 Page 46
Diya Coaching Centre for Nurses
One candle wipes out darkness…….

D. Ineffective breathing pattern related to acute increase in alveolar dead airspace


308. Which test should be added to the yearly physical of a patient? Who has recently turned 50
years old?
A. Culture and sensitivity
B. Fecal occult blood
C.Routine urine analysis test
D. Angiography studies
309. The normal range of PH in arterial blood
A.7015-7.200
B.7.25-7.3
C.7.35-7.45
D. 7.5-7.55
310. To prevent pressure on the feet of a bed bound patient with decreased tissue perfusion BEST
intervension the nurse should take is;
A. Place sheepskin under the feeds
B. Place foot cradle on the bed
C. Pad the side rails with foam tubing
D. Use only natural fiber lines
311. The nurse assists with a lambr puncture on a child with suspect bacterial meaningitis. If the
diagnosis is correct, the cerebrospinal fluid should have which of the following qualities?
A. High glucose evel
B. Low protein level
C. Cloudly or turbid appearance
D. Pink or blood –finged appearance
312.An elderly patient with serve degenerative joint disease comes to the clinic routine follow up of
pain management. The patient reports that over the past more the pain has began to I increase in
dosage of the pain mediacation. The nu recognizes that this is MOST likely due to ;
A. Drug Addition
B. Drug tolerance
C. An improvement in condition
D. Lack of efficiency of the current mediction
313.A patient had hepatitis B (HBV) and is now a chronic carrier. In plnning ... the nurse would
explain an HBV carrier would most likely be at risk for developing super infection with which other
type of hepatitis?
a. A
b. B
c. C
d. D
314. pre-operative patient has large volume ceansing enema ordered. In order faciitte the flow of
the solution into the rectum and colon, the nurse should positive the patient in the;
A. Supine position with legs flexed to chest
B. Right laterl position with left sharply flexed

Compiled by : V.Devi | 9176868900 Page 47


Diya Coaching Centre for Nurses
One candle wipes out darkness…….

C. Supine position with legs spread


D. Left ateral position with right leg sharply flexed
315. Respiratry depression is a potentially life threatening adverse effect of
A. Opiods
B. nticoagulants
C. Immune modulators
D. Non-steroidils (NSIDS
316. A child in the postictal state of seizure should show which of the following sign or symptoms?
A. Feeling sleepy or exhausted
B. Stiffness over entire body
C. Verbalizes having an aura
D. Eyes fixed in one position
317. Standards of pain management indicate that nurses
A. Administer analgestic via injection whenever possible
B. Avoid the use of the word plan
C. Screen for pain at each encounter
D. Encourage round the clock dosage of analgestics
318. The nurse observes a patient who is unable to spek when the nurse asks if the patient cn spek.
The nurse observes tht the ptient is neither coughing not cyanotic. The nurse should
IMMEDIATELY
A. Lay the patient fat before compressinmg the midstornal area
B. Insert a finger into the patient’s mouth to feel for any food
C. Stand behind the patient while performing thrusts
D. Activate the emergency call light near the patient
319. A patient required long-term antibiotic has a central line catheter inserted into the right
subclavian vein by the physician. Which of the following must be verified prior to the first use of the
catheter?
A. Blood return
B. X-ray
C. Catheter potency
D. length of catheter
320. When planning discharge teaching for a patient hospitalized for treatment of third degree burns
over 30% of the body, the nurse knows it is MOST important to include which of the following
instruction regarding the loss of lrge amounts of serum occurring with burns and the resulting loss
of immune function?
A. Wash hands frequently each day
B. wear masks while in public spaces
C. Wear supplement oxygen at night
D. Take multiple vitamin tablet each night
321. Which of the following is the most important discharge planning instruction for a patient with
mononucleosis?
A. Avoid activities that may increase injury to the spleen
B. Avoid crowded areas to prevent the spread of infection

Compiled by : V.Devi | 9176868900 Page 48


Diya Coaching Centre for Nurses
One candle wipes out darkness…….

C. Consume vitamin K rich food to decrease the risk of bleeding


D. Take an antibiotic prescribed to treat infection
322. Which of the following test measures quantity of prothrombin in blood and monitors the
effectiveness of warfarin sodium therapy and prolonged deficiencies in the extrinsic factor ?
A. Thrombin time
B. Prothrombin time
C. Partial prothrombin time
D. Activated partial prothrombin time
323. While conducting a class for expected mothers the nurse explains the differenced between true
labour and false labour contraction by indicating that the labour contractions: A. Are located mainly
in the abdomen and groin
B. Have increasing intensity.
C. Occur with increasing intervals.
D. Occur at regular intervals
324. One month after starting new medication for hypertension pt returns clinic with blood
pressure in the range. The pt admits of taking medication only when’ feeling bad’. Which of the
following actions would the nurse take?
A. Assess further determines the reason for pt’s action
B. Add new diagnosis for non compliance
C. Re- educate the pt about the importance of following his medication plan
D. Re evaluate the need for daily medication since the blood pressure is acceptable
325. Home care nurse visit p[patient who is wheelchair bound due to recent motor vehicle accident.
The pt has been sitting in the wheel chair for extended periods of time which resulted in the
development of a stage pressure sore on the right buttocks. What is the best nursing intervention?
A. Instruct the caretaker to change the position evry two hours
B. Apply hydrogel to the stage 1 pressure sore every 8 hours
C. Refer the pt to the wound care specialist for debridement
D. Encourage the pt to consume the increased amount of calcium
326. Following n open-cholecystectomy the nurse woud instruct the patient the expect to resume
normal activity.
A. 1to 2 weeks
B. 2 to 3 weeks
C. 4 to 6 weeks
D. 6 to 8 weeks
327. patient had retinal detachment surgically repaired the nurse identified the detachment would
most line be correct and unlikely to reoccur if the retina remain attached at LEAST.
. 3 days
B. 2 weeks
C. 2months
D. 3 months
328. Home care nurse visits an elderly patient who had a surgical repair for fracture. The patient is
taking opiod analgestics for pain.Today the patient comple of decresed appetites and absence of a
bowel movement for four days which of the following can be inferred.
Compiled by : V.Devi | 9176868900 Page 49
Diya Coaching Centre for Nurses
One candle wipes out darkness…….

A. Constipation related to use of opioids


B. Decreased appetite due to depression
C. Constipation due to acute pain
D. Decreased appetite due to use of opoid
329. A child recently diagnosed with sickle cell anemia is being prepared for discharge. Which of the
following statement by one of the parents would require ADDITTIONL teaching by the nurse?
A. high altitude can be necessary in the future
B. Blood transfusion may be necessary in the future
C. Strenuous physical activity should be avoided
D. Increased fluid intake minimize pain
330. 13 yrs old child is hospitalized for treatment of sickle cell crisis the nurse finds the child lying
on the side with the knees palled to words the abdomen. The parents are the bed side. The child is
crying and does not answer the nurse the parents about the child’s pain tolerance?
A. Tel the child to rest while and the nurse will return at another time for evaluation.
B. Ask the child to describe the pain it is located, and to rate it on the baker pain scale.
C. Ask the parents to medicate the patient with the medication ordered for break through pain as
soon as possible. The resume the evaluation.
D. Interview the patient about the child’s pain tolerance and usual medication requirement.
331. The nurse is entering the room of a patient who is bind. The nurse should
A. Speak before touching the patient
B. Talk to the patient using aloud tone of voice
C. Ask then patient using can be answered “yes” or “No”
D. Stand directly in front of the patient while talking.
332. patient presents to the cinic voiang concern about being exposed to hepatits (HAV) on week
ago. Upon questioning the nurse finds the patients purchased food from person recently diagnosed
with HAV. The nurse would be MOST correct when instructing the patient.
A. The incubation period is 3 to 5 weeks
B. HAV is spread by sexual transmission
C. HAV is spread by blood contact
D. The incubation periods is 2 to 5 weeks
333. While performing apre-operative ssessment on patient having arthoscopy of right knee, a
nurse examines the right leg for baseline assessment. The nurse should include all of the following
EXCEPAT
A. Position and length of leg
B. Bilateral pulse
C. Boney prominence of ankle and feet
D. Rottion of patella
334. A patient had right knee surgery and is being transferred to the post anesthesi cure unit of the
following information ESSENTIAL to discuss?
A. Pre operative weakness of the lower extremities
B. Anxiety related to inherited risk factors of surgery
C. Fear related to body image disturbance
D. Allergy to aspirin based products
Compiled by : V.Devi | 9176868900 Page 50
Diya Coaching Centre for Nurses
One candle wipes out darkness…….

335. Patient who underwent right knee arthroplasty two days ago has nursing diagnoses of
impaired mobility. The patient refuses to get out of bed ambulate due to chest pin which of the
following actions would the nurse most likely implement?
A. Medicate the patient prior to ambulation
B. Add a nursing diagnosis of non compliance
C. Let the patient rest now then try to ambulate later
D. Assess to determine the cause of the chest pain
336. After total knee replacement a patient is being discharged to home after which be will ambulate
four prong cane when. providing patient teaching regarding going up and down stairs with the
cane the first step in going up stairs.
A. Place the cane and the affected extremely up to the step
B. Place the cane and the unaffected extremely up on the step
C. Step-up on the affected extremely
D. Step-up on the unaffected extremely
337. A nurse is caring for patient who had right mastectomy 2 days go. Which of the forming is an
appropriate nursing goal for this type of surgery?
A. Accept altered body image
B. Avoids large croweds
C. Limits right arm movement
D. Perform range of motion for left arm
338. Which instruct have priority in reducing anxiety related to surgery?
A. Surgical procedure and postoperative exercises
B. Risk of infection after surgery
C. advanced direction and what is means
D. Pre-operative laboratory result and what to expect on it
339. Nurse is assessing patient who had surgery under GA. The patient’s RR is 4/mt and oxygen
saturation on 3ml/mt on O2 via nasal cannula are 84%. The nurse is awaiting the results of ABG and
anticipates that which of the following elevated.
A. Arteril O2 saturation
B. Hydrogen ion concentration
C. Partial pressure of arterial O2
D. Partial pressure of arteril carbon dioxide
340. A nurse has been visiting bed-bound patient with decreased bowel mobility it the home for
nurse that the patient is becoming incontinent of faces. The nurse evaluates the pan of care and
notes which of the following intervention would most likely beneficial?
A. An enem two times week
B. increased fiber in the diet
C. A routine bisacodyl (dulcolax suppository
D. An enema three times a week
341. Abed bound patient has a care plan with intervensions to incude re-positioning every 2 hrs. The
patient develops stage 1 pressure stage sore on the right heel. What intervention should be added to
the care plan?
A. Massage the right heel 4times per day
Compiled by : V.Devi | 9176868900 Page 51
Diya Coaching Centre for Nurses
One candle wipes out darkness…….

B. Add a trapeze to the bed


C. Float heels off bed with a pillow
D. Add a bed cradle to the bed
342. A patient is receiving from surgery using spinal annesthesia. The patient developes a spinal
headache. Which of the following nursing actions would be most appropriate?.
A. Elevate the head of the bed 30 degrees .
B. Keep the patient well hydrated
C. Limit intake of salty foods
D. Lower the temperature of the room
343. A nurse is giving discharge planning instruction to the parents of a 1 year old child with acute
ottis media. Which of the following discharge instruction taken first priority?
A. Administer antibiotics as prescribed
B. Breast feed as long s possible
C. Administer influenza vaccination
D. Avoid smoking around the child
344. Three weeks post amputation of the leg the patient is instructed to message the residual limb.
The most likely rtionle for this to
A. Provide counter-irritation for pain control
B. Prepare for prosthesis
C. Promote wound healing
D. romote acceptance of the limb’s ppernce
345. A patient receives blood transfusion for severs anemia after surgery. While evaluating the
patient the nurse finds that the patients oral temperature has began to rise from 98.2 0 f(36.80c to
11.00 F (38.30C. What should the nurse do?
A. Give the patient an antipyretic medication and continue the transfusion an ordered
B. Discontinue the IV line and restart in another site
C. Stop the transfusion, keep the vein open with normal saline, and notify the doctor
immediately
D. Use blood cooling device to cool the blood as infuses
346. The nurse is teaching patient who has just diagnosed with bacterial conjunctivitis. The nurse
should tech that the most effective way to transmission of this to other people is by
A. Putting on clean gloves before cleansing the eye
B. Taking medication as prescribe
C. Wearing guze eye patch
D. Performing hand hygiene
347. A nurse for child with celiac dises. The patient would have a permanent inability to tolerate
A. Protein
B. Gluten
C. Fat
D. Carbohydrate
348. The nurse is caring for a patient who had ligation of intracranial aneurysm via craniotomy 6
hours ago. The nurse should understand that which of the following prescribed medition will help
to minimize ICP.

Compiled by : V.Devi | 9176868900 Page 52


Diya Coaching Centre for Nurses
One candle wipes out darkness…….

A. Morphine sulphate
B. Potassium Chloride
C. Warfarin sodium
D. Basacody(dulcolax)
349. When planning discharge teching for the prent of an infnt with bronchiolites the nurse should
empharis.
A. Use of supplemental oxygen t night
B. Frequent hand washing
C. Sleeping in the supine position
D. Rice thickened formula during night time feedings
350. A nurse is caring for a child who is post tonsillectomy and adrenoidectamy. The nurse should
plan to assess which of the following complications?
A. Pulmonary hypertension
B. Hemorrhage
C. Hearing loss
D. Core Pulmonale
351. A patient has multiple clerosis and complaints of over whelming fatigue. The nurse would be
most correct in instructing the patient to
A. Conserve energy during activities of daily living
B. Increase muscle strength through aerobic exercise
C. Ignore fatigue and keep working
D. Increase early afternoon intake of caffeine.
352. While caring for an edematous patient with multiple pressure sore, the nurse asked by the
patient’s spouse to evaluate several menus, which of the following menus would be most
therapeutic?
A. Steamed carrots, milk up apple sauce
B. Tuna fish with mayo naice, boiled egg and yogurt
C. Grilled steak, baked potato and peach pie
D. Chicken noodle soup, banana and cocoa
353. When administering an oral medication to toddler, which of the following interventions should
the nurse plan to use ?
A. Depress the child’s chin with thumb to open the child’s mouth
B. Place the medication in a nipple for the child to suck
C. Give the child a small medication cup for day
D. Tell the child that the medication taste good
354. The nurse is monitoring a patient’s urine to determine hydration status. What urine colour
would indicate the best hydration?
A. Clear
B. Amber
C. Tea
D. Pale gold

Compiled by : V.Devi | 9176868900 Page 53


Diya Coaching Centre for Nurses
One candle wipes out darkness…….

355. A patient is being evaluated due to onset of paleness, shortness of breath and sensation of heart.
Palpitations which of the following component of CBC should the nurse review to determine if the
patient has anemia?
A. Leukocytes
B. Platelets
C. Erithrocytes
D. Thrombocytes
356. While a nurse is assessing a patient who reports pain due to indigestion that radiates in to the
jaw. The jaw pain is rated scale of 0 to 10. The patient reports the pain started an hour ago. The
nurse should immediately:
A. Assess the patient’s oral temperature
B. Determine what food the patient ate
C. Place the patient to reverse trendelenburg position
D. Obtain order and administer morphine sulphate
357. An elderly home bound patient is visited by the community health nurse. During evaluation
decreased skin tuegor is noted. When asked about fluids intake, the patient states that she does not
drink any fluids after lunch each day, and not wal very thirsty. The most appropriate question for
the nurse to ask is :
A. How much protein does you normally eats for dinner?
B. How much caffeine are you consuming each day?
C. Are you having trouble controlling your bladder at night?
D. Do you have enough money to buy liquids/
358. A nurse is caring for a patient who had rhinoplasty 2 weeks ago. Which of the following is an
expected outcome?
A. Oral mucus membranes dry, but pink and intact
B. Face and nose free from swelling
C. Able to make needs know. Speech therapy
D. Demonstrate throat clearing while eating.
359. A patient presents to the emergency room with complaints of eye drainage planning for
examination of the patients complaints, which of the following instruction would the nurse most
likely select?
A. Sphygmomanometer
B. Thermometer
C. Ophthalmoscope
D. Otoscope
360. A home health nurse has completed the assessment of a 72 year old patient with a gait
disturbance who will begin home physical therapy. During the interview the patient reported
significant difficulty sleeping more than 4 hours at night. Which of the following responses would
be appropriate for the nurse to make?
A. Try doing some types of exercise 2 hours before bed time
B. Drink a cup of warm tea before you go to bed
C. Make sure the bedroom is dark when u get in bed
D. A nap in the middle of the day should help
Compiled by : V.Devi | 9176868900 Page 54
Diya Coaching Centre for Nurses
One candle wipes out darkness…….

361. A nurse is caring for a 3 weeks old infant who was just admitted to the hospital. Which of the
following nursing interventions does not support this infant’s basic emotional and social needs?
A. Provide for continual contact between parents and infant
B. Activity involve parents in caring for the infant
C. Keep the infant’s environment quiet, dim and free of sensory stimulation
D. Foster infant sibling relationship as appropriate
362. A homecare nurse visit a patient who is discharged from a hospital after a treatment of
urosepsis. Which of the following post discharge normal laboratory results indicate best desired
outcome?
A. WBC count
B. Hematocrit
C. Platelet level
D. Potassium level
363. a nurse visits a patient which is 37 weeks pregnant and asking for information about breast
feeding verses feeding prepared infant formula. A beneficial reason to breast feeding includes:
A. Readly available and economical
B. Keep a baby full longer
C. Lager curds than cows milk and therefore is easier to digest
D. Encourage greater deposits of subcutaneous fat in an infant
364. When implementing a feeding schedule for a full term 2 weeks old infant, the nurse should
expect the infant to be fed
A. 2-4 times per day
B. 6-8 times per day
C. 10-12 times per day
D. 14-16 times per day
365. N home care nurse makes a follow up visit to a patient who had shingles a month since the
onset. The patient pain level is 6 on a scale of 1 to 10, where 1 is no pain and 10 is greatest pain. Two
weeks ago the pain level was at 7. The patient is treated with pain medication as needs, and skin
vesicles are completely resolved without any scaring. The patient’s condition has:
A. met the expected outcome
B. Partially met the expected outcome
C. Has not improved
D. Has worsened
366. The nurse is in public area of the health care facility when an adult falls to the floor.Which of
the following actions should the nurse take next?
A. Open the airway
B. Determine unresponsiveness
C. Activate the emergency call system
D. Obtain the Automatic Electronic Defibrillator(AED
367. When caring for a patient who is receiving anticoagulant medication, the nuse must monitor
the patient for signs of
A. Skin breakdown
B. Bleeding
Compiled by : V.Devi | 9176868900 Page 55
Diya Coaching Centre for Nurses
One candle wipes out darkness…….

C. Pain
D. Confusion
368. A patient is being prepared for a right breast biopsy under general anesthesia. The patient asks
the nurse about the surgiuacal scar and possible postoperative complications. Which of the
following actions would be appropriate for the nurse to take?
A. Review of postoperative risks with the patient
B. Notify surgeon about the patients questions
C. Complete the patient’s preoperative
D. Show the patient photos of breast surgical scars
369. A patient with bowlegs due to abnormal bone formation and deformities has a calcium level of
7.5mg/100ml. Which of the following foods would the nurse most likely instruct the patient to add
to a diet?
A. Organ meals
B. whole grains
C. Egg yolks
D. Lean meats
370. A patient has just diagnosed with hypothyroidism. Which of the following instructuions is
correct?
A. You will need to take thyroid hormone replacement therapy for your entire life
B. You will need to take thyroid hormones replacement therapy until hypothyroidism
C. You will need to take thyroid hormone for 2 months
D. you will need to take thyroid hormone for one year
371. The stages dying as identified by Dr. Elizabeth
A. Anger, depression, bargaining, denial
B. Bargaining, denial, acceptance, depression
C. Denial, anger, Bargaining, depression, acceptance
D. Depression, denial, anger, bargaining, acceptance
372. A co-worker informs that the nurse about experiencing increased level of work associated with
daily job responsibilities to help cope with professional stress, the nurse should encourage the co
worker to.
A. make a list of unfinished tasks
B. Complete complex mental tsk before physical tsks
C. Acknowledge daily accomplishments
D. Spend time with colleague away from work
373. A nurse is caring for a POP patient who is on subcutaneous low dose heprin. When
administering an injection on the abdomen the nurse avoids the umbilical area because of the
possibility of
A. Entering a large blod vessel
B. Causing increased pain
C. Precipitating hyper ventilation
D. Umbilical infection
374. A patient with conjunctivitis reports the presence of photophobia. The nurse should teach the
patient to
Compiled by : V.Devi | 9176868900 Page 56
Diya Coaching Centre for Nurses
One candle wipes out darkness…….

A. Avoid touching the eye


B. Use sterile guaze to remove the drain
C. Darken the room
D. Rest in the prone position
375. During surgery the patient had following intake and output, IV fluid 650cc, IV antibiotic 50cc, 1
unit of packed RBC 350cc, NG out put 120cc, blood loss 85cc and urinein FC 240cc. What is the
[patient’s total input?
A. 650cc
B. 700cc
C. 950cc
D. 1050cc
376. A nurse is performing assessment on patient’s nutritional status. Which of the following would
be the best measure of patient’s recent nutritionl status. Which of the following diagnostic test
would be the best mesure of the patient’s recent nutritionl status with a half life of 2-3 days?
A. pre albumin
B. Haemoglobin
C. Albumin
D. 24hr urine creatine
377. A nurse is caring for a patient with pneumonectomy 2 days ago for lung cancer. Which of the
following would indicate that the patient is progressing towards discharge goals?
A. Cough productive of serosangeous fluids
B. 1+ pretibial edema
C. Nas after completting bed bath
D. Frequent ventricular contractions
378. The nurse is caring for a ptient with parkinsons disease. Which of the following is an expected
outcome related to nursing diagnosis of constipation related to diminished motor function,
inactivity and medications.
A. The patient will use a laxative every other day
B. The patient will have a soft bowel movement daily.
C. The patient will report minimal pain with bowel movements.
D. The patient will limit intake of complex carbohydrates.
379. A community health nurse assesses a 68 year old patient who lives in a group home The patient
reports decresed apetite after transferring to the group home because the food taste too bland. What
type of the data the nurse collecting from the above information?
A. Analytical
B. Derived
C. Objective
D. Subjective
380. The homecare nurse is providing wound care for a patient. The nurse evaluates the wound nd
notes the presence of granulation tissue in the wound bed. This observation represents which phase
of wound healing.
A. Maturation
B. Inflammation
Compiled by : V.Devi | 9176868900 Page 57
Diya Coaching Centre for Nurses
One candle wipes out darkness…….

C. Proliferation
D. Finalization
381. A nurse is caring for a 3yrs old child with a fractured arm. Which of the following interventions
is the most appropriate for pain management?
A. Administers analgesics when necessary
B. Assess pain once a shift
C. Anticipate pain and intervee early
D. Encourage the use of self quieting techniques
382. which statement by the patient with hyperlipedermia shows a basic understanding of the
disease and its treatment?
A. Exercise has no effect on cholesterol levels
B. Hyper lipidemia is usually symptomatic until significant target organ dmage is done.
C. HDL cholesterol level of greater than 60mg Idl increases the chance of coronary artery disease
D. Chloesystyramine should be taken in the morning with other medications
383. A doctor has ordered a intramuscular injection for a 6 month old infant after her IV infiltrated.
Because infant have under developed muscles, the nurse should not administrates the injection in to
which muscle?
A. Vastus laterals
B. Rectus femoris
C. Ventrogluteal
D. Gluteus maximus
384. A nurse caring for a patient with cute pulmonary edema observes that the patients cough
produces white frothy and that the patient is extremely dyspneic. The patient has inspirations and
expiratory wheeling on auscultation of the lungs. The immediate objective of treatment is to
A. Improve oxygenation
B. Decrease anxiety
C. Improve tissue perfusion
D. Decrease risk for spirtion
385. When discussing dietary choices with a patient who is on heparin therapy the nourse should
teach the patient that which of the following foods may increase clotting time?
A. Grape fruit
B. Orenges
C. Bananas
D. Red grapes
386. A 2 month old child in the emergency department has projectile vomiting after feeding. The
vomiting is non bilious containing milk nd gastric juices. Immediaately after vomiting the child tries
to feed again. The nurse palpates the child’s bdomen during feeding and notes a firm area to the
right of the umbilicus in the ippes right quadrant. Which of the following is consistent with the
history?
A. Hyper tropic pyloric stenosis
B. Hirchsprungs’s disese
C. GERD
D. Trachoesophgel fistula
Compiled by : V.Devi | 9176868900 Page 58
Diya Coaching Centre for Nurses
One candle wipes out darkness…….

387. A patient undergoing cancer treatment has developed acute hyper calcemis with signs of
weatness, nausea and vomiting. Which of the following would the nurse anticipate to be the initial
treatment?
A. Thiazide diuretic
B. Intrvenous normal sline(0.9% NACL)
C. A pttasium supplement
D. Broad-spectrum antibiotic
388. The patient is receiving mechanical ventilation set at fraction of inspired oxygen (FI02) 100%.
The nurse should understand that which of the following can improve the patient’s oxygenation?
A. Adding positive end expiratory pressure(PEEP
B. Placing the patient in Trendlenbueg position
C. Increasing the FI 02
D. Suctioning the patient hourly
389. Which of the following nursing diagnosis takes priority for a patient after gastrointestinal
surgery?
A. Impared skin integrity related to surgical incision
B. Constipullation related to surgery
C. Risk for infection related to surgical incision
D. Acute pain related to surgical incision
390. Marijuana is an example of a drug classified as schedule
A. C I
B. C II
C. C III
D. C-IV
391. A patient with a weight loss of 12 in 60 days has a nursing care plan written interventions
including offering a dietery supplement three times per day>
After 2 weeks, the patient has had another 1% weight loss. The patient indicates no likely the
supplements. The nurse should
A. Continue the pln of care as written
B. Replace the supplement with high calorie food the the patient likes
C. Encourage the patient to drink supplements
D. Offer smaller amounts of supplement more frequently
392. The nurse is caring for a patient with magnesium toxicity. Which of the following clinical
manifestation should the nurse anticipate?
A. Parasthesia
B. Decreased deep tendon reflexes
C. Cardiac palpitations
D. Decreased cardiac output
393. A ptient returning from 3 hours shoulder repair with general anesthesia is being transported
from the operating room (OR) to the post anesthesia care unit(ACU). The nurse knows that the
patient is at high risk for injury related to residual annesthesia. During this timeperiod the patient is
at lwest risk for
A. Airway obstruction
Compiled by : V.Devi | 9176868900 Page 59
Diya Coaching Centre for Nurses
One candle wipes out darkness…….

B. Vomiting
C. Impared circulation
D. Fluid volume deficit
394. For a patient schedule fr a total pancreatomy the nurse would be instruct the patient that hte
procedure most likely cause
A. Pancreatic ascites
B. Chronic pancreatitis
C. Diabetics mellitus
D. Diabetics incipedus
395. The nurse is assessing an infant for possible deafness. Which of the following automatic reflexes
would the nurse most likely check to best determine whether the child has a serious hearing
problem?
A. Blinking
B. Vertical suspension
C. Moro
D. Perez
396. The nurse is teaching a group about aerobics exercise. When discussing the trget heart rate for
exercise, the nurse should state that this is calculated by
A. Counting the number of the heart beats during exercise for 6 sections, then multiply this number
by10
B. Subtracting the chronological age from the number 220
C. Counting the number of heart beats during exercise for 10 seconds, then multiply by 6
D. Subtracting chronological age from 240
397. While performing an assessment on a post surgical patient 2 days after surgery, the nurse notes
shallow and rapid respirations. What should the nurse do next?
A. Assss the patient for pain
B. Obtaian an order for supplementary O2
C. Elevate the head of the bed
D. Place a warmed blanket on the atient
398. A patient is receiving intravenous fluids of a ration of 125ml/hour. What vlume fluids will the
patient receive during an 8 hours shift?
A. 1500ml
B. 1 litter
C. 1.5 litter
D. 500ml
399. A patient has a history of severe, uncontrolled epistaxis. The patients blood pressure and
patient count are normal. The nurse should teach the patient to:
A. Sleep with the head elevated on at least two three pillows
B. Apply firm pressure to the nostrils four times a day
C. Use a cotton tipped applicator t apply a water soluble lubricant to the nasal spectrum twice daily
D. Minimise the intake of caffeine while using the intake of fluids rich i vitamin K
400. A nurse is caring for an infant with respiratory distress syndrome. Which of the following
nursing interventions is appropriate?
Compiled by : V.Devi | 9176868900 Page 60
Diya Coaching Centre for Nurses
One candle wipes out darkness…….

A. Measure oxygen saturation level once a shift


B. Suction frequently for 30-45 second each time
C. Monitor for symptoms of hyper glycemia
D. Maintain infant temperature b/n 367 degree and 37.8 degree celcious (970 and 980 F)
401. A nurse is caring a patient who had a left mastectomy with lymph node removal seven days
ago. The patient asks about exercises to regain function of the left arm. Which of the following
activities would be most appropriate?
A. Walking fingers up the wall
B. Using five pound weights
C. Rhythmic clapping
D. knitting with large needle
402. What occurs during cadiogenic shock and results in inadequate tissue perfusion?
A. increased resistance of arterial vessels
B. Decreased effectiveness of the heart as pump
C. Increased shunting of critical blood flow to heart
D. Decreased capacity of the venous beds
403. The nurse is caring for child admitted with viral pneumonia. Which of the following nursing
diagnoses should receive priority?
A. Nutrition altered less than body requirements
B. Ineffective airway clearance
C. Fluid volume deficit
D. Risk of injury
404. A child has injested an entire bottled acetaminophen(Tyenol). Which of the following organs is
affected?
A. Liver
B. Brain
C. Kidneys
D. Gallbladder
405. A home care nurse make a follow up vist to a patient who recently suffered a cerebro vascular
accedent. The patient is mobile and able to perform activities of daily living. However, the patient
has not been sleeping and has lost weight due to lack of apetite. The patient also feels overwhelmed
with sadness. Which of the following is the most appropriate evaluation?
A.. Patients progress is unexpected and no further intervention is necessary
B. Patient needs referral to a nutritionist.
C. Patient needs intervention for depression
D. patient needs sleeping medication
406. A patient is seen in the emergency department with complaints of angina. Nitroglycerin
(Nitrostat) is ordered by the physician. This medication is to be administered via which of the
following routes?
A. Intrademal
B. Buccal
C. Parental
D. Topical
Compiled by : V.Devi | 9176868900 Page 61
Diya Coaching Centre for Nurses
One candle wipes out darkness…….

407. The nursing is teaching a group of patient about hepatitis A (HAV). The nurse should state that
HAV is mainly transmitted via
A. Blood contact
B. Food
C. Sexual activity
D. Salvia
408. A child was recently diagnosed with spastic cerebral palsy. Which of the following statements
by the parent would indicate to the nurse that parent understand teaching about this illness?
A. Full recovery is possible
B. This illness should not progress
C.cerebral palsy is a hereditary disease
D. Surgery cn some times improve walking
409. patient hospitalized with crohn’s disease has developed fever, increased respiratory rate
increased heart rate, chills, diaphoreses, and used abdominal discomfort. The nurse knows that
patient has most likely developed
A. Intestinal obstruction
B. Intestinal parasite infection
C. intestinal perforation
D. Asitis
410. A child is admitted to the hospital with dehydration. The nurse should give priority to which of
the following nursing dignoses?
A. Anxiety related to hospitalization
B. Fluid volume deficit related to vomiting
C. Imbalance nutrition less than body requirements related diarrhea
D. Risk for infection related to presence of invasive lines
411. The nurse is caring for patient with DVT. The patient’s heparin sodium infusion has been
discontinued and the patient is receiving prescribed warmfarin sodium (Coumadin. The nurse
advice the patient that which of the following needs to be continued?
A. Daily CBC
B. Laboratory test for partial thromboplastin time (PPT
C. Strict bed rest
D. warming elasticized support stockings
412. hen teaching the parents of neonate with spina fibida tequniques to promote bladder emptying
the nurse reviews atechnique in which firm gentle pressure is applied to the abdomen pressed
towards the symphysis pubis. This method is known as
A. Credes
B. Intermittent
C. Foley
D. prophylactic
413. A 50 years old patient is being admitted to the hospital in a vegetative stte of unknown etiology
what is the priority nursing diagnosis?
A. Risk for impaired skin integrity
B. Impired swallowing
Compiled by : V.Devi | 9176868900 Page 62
Diya Coaching Centre for Nurses
One candle wipes out darkness…….

C. Altered cerebral tissue perfusion


D. Altered thought processes
414. Prior to administrating an enema, the nurse will assist the patient to assume what position
A. Prone with pillow under the knwws
B. Left side with right knee flexed
C. Right side with left knee flexed
D. On back with head of bed flat
415. A nurse interviews a patient, recently admitted to long term care facility , to obtain information
on the ptient’s health perception. The nurse encourage the effective in this situations?
A.Analytical
B. Focused
C. Closed
D. Open-ended
416. When selecting activities to help develop child’s fine motor skills, which of the following
would best meet this goal?
A. Sorting cardboard objects that are in different shapes
B. Singing while turning the pages of a book that plays music
C. jumping rope
D. Riding a three-wheeled bicycle
417. 6 years age a patient weighed 73 kg (161 pounds). During the current clinic visit the nurse note
the patient has an unintended wt loss. This wt loss over 6 months would be considered clinically
significant as soon as it reaches the point of being more than a
A. 5%loss
B.8%loss
C.10%loss
D.20%loss
418. child with a diagnosis of tetralogy of fallot is scheduled to be discharged from the hospital the
nurse planning discharge education should instruct the care gives that during a hypercyanotic spell
the position most likely to benefit the child is
A. Supine
B. Side-lying
C. Prone
D. Knee-chest
419. A child is treated for possible acetaminophen (Tylenol) overdose. The child is currently stable
with normal vital signs. Which of the following organ function system would be most affected
A. Liver
B. Stomach
C. Lungs
D. Heart
420. The nurse is caring for a patient with stage III pressure ulcer to the coccyx. There days after
initialling the plan of case, the nurse observes that the ulcer has hard black crust covering the center
of the ulcer. The nurse should understood that this indicates
A. healing
Compiled by : V.Devi | 9176868900 Page 63
Diya Coaching Centre for Nurses
One candle wipes out darkness…….

B. Need for debridement


C. Inadequate nutrition
D. Infection
421. To limit drug interactions, the nurse should advise the parent of chronicallt ill child to
A. Refers to the medictions by the generic name
B. Teach the child the name of all medications prescribed
C. Give all medications one hour apart
D. Get all prescriptiuons filled at the same pharmacy
422. The nurse receives an order to obtain an artetial gas (ABG specimen on a patient. The nurse will
use the redial artery to obtain the specimen. Which of the following will the nurse assess before
puncturing the redial artery?
A. llen test
B. Prtial pressure of rterial oxygen
C. Partial crbon dioxide
D. Prothrombin time
423.For an infant with hydrocephalus, anurse should plan to monitor for what sign or symptom of
increased intracranial pressure?
A. High pitched shrill cry
B. decrease in systolic blood pressure
C. Depressed frontanelle
D. Increase in respiration
424. During a surgery requiring general anestasia, the patient’s heart stops. Ventilations using the
endotracheal tube (ETT are started with an ambu bag. Which of the following compression to
ventilation rates is correct?
A. 1 to 2
B. 15 to 2
C. 5 to 2
D. 1 to 2
425. The nurse is caring for apatient scheduled for left arm amputation due to bone crcinom.
Dequate ssessment and management of preoperative pain will result in
A. Decreased phntom limb sensation
B. Increased ROM after surgery
C. Decresed depression after surgery
D. Decreased likelihood of cancer recurrence
426. A 34 years old quadriplegia patient resides at home with his wife. In order to prevent
contractures of all extrimities, the community care nurse will instruct the patient’s wife in the
performance of
A. Active ROM exercises
B. Passive ROM exercises
C. Active assistive ROM exercises
D. Resistive ROM exercises
427. A 7 years old child is briught to the emergency room with complaints of the sick for 3 weeks
with sore throat, cough and muscle pain. Upon examination, the nurse notes alow-grade fever,
Compiled by : V.Devi | 9176868900 Page 64
Diya Coaching Centre for Nurses
One candle wipes out darkness…….

shortness of breth, and a wheeze on uscultation child lives with prents, 6 siblings and grandfather in
a 3 bedroom house. Based these findings, which of the following diagnosis would be most likely
A. Staphylococal pneumonia
B. Pneumocystis crinji pneumoni (P
C. Bronchiolities
D. Mycolsm pneumonia
428. A patient comes to the emergency department complaining of severe crush substernal pain that
radiates to the left arm & jaw. The patient is diaphoretic and with cool clammy skin. The patient is
diagnosed with acute myocardial infarction nursing diagnoses would be decreased cardiac output
related to
A. Structurl factors (Incompetent valves
B. Impaired ventricular expans
C. Impaired contractility
D. Fluid volume deficit
429. After a hearing restoration operation, a patient has no signs of complication soon recovers
which of the following is an executed outcome 5 days after the restoration surgery?
A. Regain full hearing
B. Minimal facial nerve paralysis
C. minimal urinary incontinence
D. Ambulate without difficulty
430.When teaching apatient how to use a cane after a CVA. The nurse should sure the patient
A. Uses the cane on the unaffected side
B. Advances the cane simultaneously with affected limb
C. Holds the cane away from the body
D. moves the cane past the toes of the affected limb
431. A home care nurse visits a patient diagnosed with diabetes mellitus whose current glucose level
rages from 15mgldl to 200mgldl. The patient has not be to self administration prescribed bed insulin
and complains of blurred vision and a inability to read the marking on the proper insulin dosage.
Which of following referrals would be most beneficial to the patient?
A. A dietician
B. An endocrinologist
C. An ophthalmogist
D. A physical therapist
432. The doctor has orders the patient to be on 1 to 3 liters of oxygen using a canula at all times. The
home care nurse notes the O2 is currently at 2 as/min O2 saturation SO2 residing is currently 85%
and the partial pressure (Paco2 I normal limits. Based on an evaluation of this information, which of
the follow actions would the nurse most likely perform?
A. Decrease the O2 to IL/min & monitors O2 saturation
B. Continue the O2 t 2L/min & monitors O2 saturation
C. Increase the O2 to 3 L/min & monitor O2 saturation
D. Continue the monitor O2 saturation & call the doctor for new order
433. A child is treted for bacterial meningtits with an intravenous antimicro agent. Which of the
following Best indicates effectiveness of the treatment?
Compiled by : V.Devi | 9176868900 Page 65
Diya Coaching Centre for Nurses
One candle wipes out darkness…….

A. Incresed appetite
B. Temperture 37.2 degree celcious (99
C. Episodes of apnea
D. increased icp
434. A patient with (GERD) is to start taking prescribed omeprazole (prilsec. The nurse would
instruct that patient to take the mediction
A. 30 to minutes before meals
B. 90 to 12 minutes before mels
C. with apple sauce
D. with milk
435. A patient recently diagnosed with multiple sclerosis has been taking the following prescribed
medications beclofen (Lioresal) diazepam(valium Amantadine(symmetrel and phenytoin (dilantin)
when the patient presents with complaint of ftigue, the nurse should address the dosage and
frequency of which medication?
A. Baclofen (Lioraseal
B. Diazepam (valium)
C. Amantadine (symmetrel)
D. Phenytoin (Dilantin
436. A patient is receiving ablood transfusion post-surgery. The nurse is abut to take the frst set of 15
minutes vital signs when the patient states “I am freezing cold and my chest feels tight what should
the nurse do next?
A. Stop the transfusion, keep the vein pen with normal saline and notify the ordering physician
B. Continue the transfusion, start normal saline, and re check vital signs in 1 minute
C. Slow the transfusion to hlf the rate and administer acetaminophen (tylenon) per standing order
D. Stop the transfusion, bring the patient ablanket and notify the physician
437. nurse is caring for a patient who is receiving from radio frequency abletion of matastic liver
tumors. The patient has been laughing and visiting with friends throught out he shift. Shortly before
the end of the nurse’s shift, the patient uses the call bell to request medication. The patient rates the
pin as a 9 on scale of (no pain) to 10(severe pain). The nurse observes that the patient is smiling and
respirations are evenand unlabored. Thenurse should
A. Wait before administering the pain medication
B. Administers pain medication as ordered and assess for effectiveness
C. Review the patient’s chart for any history of drug abuse
D. remind the patient to request analgesic before the pain is severe
438. A patient is being treated for asthma in the emergency department. An ABG is ordered to
evaluate the patient’s oxygenetaion prior to treatment. What is the name of the procedure shown in
the graphics
A. Allen test
B. Arterial puncture
C. papanicolaou test
D. angiography

Compiled by : V.Devi | 9176868900 Page 66


Diya Coaching Centre for Nurses
One candle wipes out darkness…….

439. A 51 yrs old pt in hospital clinic is scheduled for colon biopsy. The pt speaks a different
language than the hospital staff, but does understand simple communical in the language of staff.
When conducting pt education prior to the procedure the nurse should plan?
A. Write all communication and avoid speech
B. Raise the volume and pitch of the voice
C. obtain an interpreter
D. smile and nod frequentlty
440.The following medictions are ordered for a pt who had a rigid leg amputation oxycodone sing
every 4 hrs as needs and morphine 5 mg every 4 hrs as needed. The nurse dministered oxycodone 2
hrs ago but the pt report pain rated 8 on a scale of (no pain) to 10 (serve pin as the dressing change
begins. Vital signs are Bp-169/98, HR-112, RR-22, T-36.7 C. After evaluating the effectiveness, of the
pain medication What ct ion should the nurse take?
A.Administer is additional oxycodone 5 mg
B. Administer morphine 5mg
C. Change the dressing quickly
D. Encourage deep breathing
441. child is dmitted with temperature of 38.5 degree celkcious(103.3 degree F loss of appetite and
the nurse observe several joints and red, swollen, warm and render tc touch. Non pruritic rash is on
the child’s trunk laboratory test results include on ESR, a positive proteien nd n elevated WBC. The
nurse should initiate the plan of care for
A. CHF
B. Meningitis
C. Rotovirus
D. cute rheumatic fever
442.A nurse is assessing the pheripherl circulation of the patient’s extremities. The chart indicates
the patient has edema in both lower extrimities. Which of the following assessment tequenics would
be the nurse likely use to assess for this?
A. inspection and auscultation
B. Inspection nd papation
C. palpation and percussiod
D. Percussion and auscultation
443. a child is admitted with a temp of 38.5 c loss of appetite and malaise. The nurse observes severl
joints are red, swollen warm and render. A nonprurritic rashnis on the childs trunk. Laboratory
result shows elevated ESR, +c C reactive protein and an elevated WBC. The nurse should initiate the
plan of care for.
A. CHF
B. Meninggitis
C. Rotavirus
D. Acute rheumatic fever
444. nurse is carring for a hospitalized diabetic patient with advanced peripheral neuropthy. Which
of the following nursing ction is the most important?
A. moisturizing the skin with lotion ech day
B. Ensuring that foods are not too hot
Compiled by : V.Devi | 9176868900 Page 67
Diya Coaching Centre for Nurses
One candle wipes out darkness…….

C. facing the patient when speaking


D. Assessing the heels for breakdown
445. A patient in a long term care facility is in persistent vegetative state with a high contracture of
right arm and hand. What is the best goal over the next 90 days for this patient related to nursing
diagnosis of impaired mobility?
A. Develop no further contractures
B. wear an arm and hand splint
C.Have no pain related to contracture
D. Have the contractures resolved
446. A postoperative patient has the nursing diagnosis of ineffective airway clearance. To assess for
airway clearance the nurse should check all the following except.
A. Heart rate
B. Skin colour
C. Respiratory rate
D. Breath sounds
447. the ability of the test to correctly identify those individuals who have disease is known as
A. Specificity
B. Sensitivity
C. Incidence
D. Predechive value
448. The facilitate self-care for a 2 years old child with spastic cerebral palsy the nurse should
recommend
A. Placing straws in to beverage containers
B. Obtaining eating utensils that have large handles
C. Purchasing shoes that have an open heels area
D. replacing zippers on clothing with mental snaps.
449. A 21 years old female is being discharged after 2 day admission for PID. Hich statement best
identifies the patients understanding of follow-up care for PID?
A. My sexual partner needs to be treated with antibiotics
B. Its OK to resume sexual relation now
C. I need to inform any sexual partners. I have had in the past 3 days that I had PID
D. In order to prevent getting PID I need to continue to take birth control pills
450. A health 2 years old child is brought to the community health clinic for routine check up. At
this visit the nurse should administer the following vaccine
A. Rot virus
B. Hepatitis B
C. None of this time
D. Varicella
451. During an evaluation at a community clinic the patient completes the medical history. Which of
the follow is NOT risk factor for an cute MI?
A. Coronary artery disease
B. smoking
C. Hemophilia
Compiled by : V.Devi | 9176868900 Page 68
Diya Coaching Centre for Nurses
One candle wipes out darkness…….

D. Hyperlipidemia
452. Which of the following is the most common type of crdiomyopathy in children and is treated
with medications such as digoxin (Lbixin nd wrfarin (Coumadin)
A. Hypertrophic
B.Dilated
C. Restrictive
D. Distolic
453. The responsibility for teaching patient how to take medications safetly when they are
discharged from the hospital belongs to the
A. Physician
B. Detitian
C. Therapist
D. Nurse
454. A nurse is discharging a patient after hospitalization due to myocarditis. Which of the
following statements should be included in discharge teaching?
A. There is usually some residual hart enlargement
B. May resume previous activities as before hospitalization
C. Avoid immunizations against infections disease
D. Rapidly beating hart is a common side effect f the illness and is not dangerous
455. A nurse is assessing a 4 month old formula fed infant . The parent repots the infnt has been
irritable, crying excessively, not sleeping well & vomiting GERD is expected. What nursing
intervention should the nurse expect to teach the parent?
A. Place the infnt in an infant seat after eating
B. Give large frequent feedings
C. position the child in a swing
D.Thin formul with wters
456. An adult arrived at the outpatient facility due to the onset of chest pain the patient suddenly
falls to the floor and is unresponsive . What action should the nurse take next?
A. Active emergency call system
B. open the patient’s airway
C. Check for croid pulse
D. administer 2 rescue breaths
457. A patient suffered a head truma which resulted in a nasal fracture requiring surgical
intervention.Which of the following nursing diagnoses would most likely be a problem this patient?
A. Delyed surgical recovery
B. Impired gas exchange system
C. Ineffective breathing pattern
D. Risk for periopertive positioning injury
458. After administering inhaled corticosteroids to hospitalized child with asthma, the nurse plans
to have the child rinse the mouth nd gargle with water. The nurse knows the rationale for this action
is prevension of
A.Tooth decay
B. Oral candidiasis
Compiled by : V.Devi | 9176868900 Page 69
Diya Coaching Centre for Nurses
One candle wipes out darkness…….

C. Dehydration
D. Hypertrophy of the gums
459. The nurse is assessing a patient who is 2 weeks postoperative a kyphoplasty of 12 & L3. The
patient has been partcicipating in physical therapy and has been doing daily stretching and
strengtherning. Which of the following would
A. reports pain in legs while sitting
B. Urinating every 2 hrs while awake
C. Fatigue after performing activities of daily living
D. Ambulates outdoors without assistive device
460. When a child is brought to the emergency department with acute epiglottis, which of the
following nursing diagnosis should receive priority
A. Ineffective airway clearance
B. activity intolerance
C. Fluid volume deficit
D. Impaired verbal communication
461. The nurse is reviewing the medication of a patient who is scheduled for a CABG in 3 days.
Which of the following medictions must be discontinued t least week prior to surgery?
A. Digorcin (Lanoxin
B. Furdsemide (Lasix)
C. Propanalol Hydrochloride (Indeeal)
D. Warfarin Sodium (Coumadin)
462. A patient with pneumonia experience ineffective airway clearance related to the presence of
secretions secondary to infection: O2 saturation is 89% on room air. Which of the following nursing
interventions takes priority?
A. Deliver O2 with humidity
B. Encourage fluid intake
C. assist patient in to position of comfort
D. Inspect patient in to position of comfort
463. A nurse is assessing a 5 month old infant. The parents state that infant is irritable, crying
excessively, vomiting formula (not projectile), acting and stiffening. Based on this assessment, what
diagnosis should the nurse anticipate?
A. Esophagel atresia epith tracheoesopha geal firstula
B. Gastro esophageal reflux
C. Hirschsprung’s disease
D. Celiac disease
464. A patientpresents at the clinic with weight loss and complaints of troble seeing at night. The
nurse also observes numerous teeth with decay. Upon leading that the patient has a vitamin
deficiency. Which of the following foods would the nurse likely instruct the patient to ass to diet?
A. Cheese and breads
B. Liver and milk
C. Fish and rice
D. Fruits and vegetables

Compiled by : V.Devi | 9176868900 Page 70


Diya Coaching Centre for Nurses
One candle wipes out darkness…….

465. While providing discharge teaching for the parents of a child newly diagnosed with cystic
fibrosis the nurse includes teaching regarding the role of salt in the disease. Which of the following
statements by the parent indicates the need for further teachings?
A. salty foods may be eaten on occasion
B. My child does not need to restrict salt intake
C. salt is lost more rapidly in hot weather
D. salt replacement should occurs every day
466. A patient visiting the clinic `1 days after sinus surgery for check up, complies of bad taste in the
month. When the nurse smells foul odor while examining the patient mouth the nurse suspects the
patient have
A. pulmonary decompensation
B. Hemorrhage
C. Aspiration
D. Infection
467. A patient is scheduled for a total hip arthroplasty. The preoperative nurse review the chart and
notes the following serum potassium level of 2.8mfgfq/1AB positive blood type, and elevated ST
segments on the ECG. Which of the following would be the most appropriate action for the nurse to
do next?
A. report abnormal diagnosis results to the surgeon
B. Review the patient consent for the surgical procedure
C. Educate the patient on the risk factors and side-effects
D. Ensure that the patient has a post-surgery physical therapy order
468.Which of the following discharging planning instructions takes priority in patient with
congestive heart failure
A. Maintaining a low cholesterol, low sodium and low potassium diet
B. Recognizing signs & symptoms that requires immediate medical attention
C. The importance of remaining physically active
D. The importance of drinking plenty of fluid
469. Following ocular surgery the nurse establishes care interventions to include orienting the
patients to new changes in environment and supervising the patient’s ability to feed themselves and
perform self-care activities. Which of the following nursing diagnosis do these activities support?
A. Activity intolerance
B. Impaired envioirnmetal inter[retation syndrome
C. Disturbed sensory perception
D. Risk for automatic dysreflex
470. During the immediate ostoperative period, a patient reveals an O2 saturation level of 91% The
nurse should
A. Position the patient on the left side
B. Administer supplemetal oxygen
C. Continu to provide supportive care
D. Lower the temperature of the room
471. Which of the following goal take priority when recovering from GA in post anesthesia care
unit?
Compiled by : V.Devi | 9176868900 Page 71
Diya Coaching Centre for Nurses
One candle wipes out darkness…….

A. Thermo regulation
B. elstic skin turgor
C. Patient airwy
D. patient voids freely
472. A patient is to receive heparin sodium 500 unit subscutaneous on call to the O2 prior to
administering this medication the nurse advanced patient that this will help prevent
A. Infection
B. Atelectasisc
C. thrombosis formation
D. Positioning injuries
473. When administering an IM injection to an infant which of the following site is appropriate for
the nurse to use?
A. rectus femorius
B. Deltoid
C. Dorso gluteal
D. ventro gluteal
474. A ptient is admitted to the medical unit with a diagnosis of fluid volume de and hypotension.
Which fluid would the nurse expect the doctor to order?
A. .9% sodium chloride
B. .45% sodium chloride
C. Dextran in NS
D. 5 % sodium chloride
475. The nurse is discussing HIV with a group of high risk patients. The nurse should state that this
virus is found most commonly in which of the following body fluid?
A. Blood
B. Saliva
C. Breast milk
D. Vaginal secretion
476. A parent is concerned their year old child has 203 kg (5 lb) over the past 2 weeks and has been
urination up to 30 times per day. The child also seems to be eating and drinking constantly. Which
test would be most helpful in evaluating the child’s condition?
A. Chest X-ray
B. Complete blood count
C. Body fat analyisis
D. Bood glucose level
477. A patient has been transferred to the medical unit following a parathyroidectomy. Surgery was
performed under general anesthesia and the patient’s diet may advance as tolerated. The patient
requests a sip of apple juice. The nurse should first assess the patient’s ?
A. Skin turgor
B. Cough reflex
C. Lung sounds
D. Bowel sounds

Compiled by : V.Devi | 9176868900 Page 72


Diya Coaching Centre for Nurses
One candle wipes out darkness…….

478. The nurse sustains a needle stick injury after administering an intramuscular injection to a
patient. It is recommended that the nurse be tested for human immune deficiency virus (HIV)?
A. Immediately with repeat test in 6 weeks
B. If the patient refuses HIV testing
C. If the patient has symptoms of HIV infections
D. A month after taking prophylactic anti virals
479. A parent brings their teenage child to the paediatric office. The parent reports that the patient
frequently complains of abdominal bloating and stomach pain after eating and also has a chronic
sore throat. The patient’s lab shows hypokalemia. Which of the following diagnosis should the
nurse anticipate?
A. Anorexia nervosa
B. Bulimia
C. Morbid obesity
D. Impulsive behaviour
480. A Urine analisis is best evaluated for accurate results if specimen is analyzed within?
A. 1 hour of collection or refrigerated until analised
B. 1 hour of collection or eft at room temperature
C. 2 hour of collection
D. 4 hour of collection
481. The nurse has started intravenous fluid therapy on a child. Which of the following action is
appropriate?
A. Using paddedarm board only if the child is active
B. Checking the site atleast once every 2 hrs
C. Determining the total volume infused every 4 hour
D. Using an infusion pump to provide controlled rate of infusion
482. During the assessment phase of a preoperative interview, the patient reports feeing nervous.
The patient conveys to the nurse that a parent died in surgery due to malignant hyperthermia. To
whom the information be most pertinent?
A. PACU nurse
B. Scrub nurse
C. Anesthetic team
D. Charge nurse
483. A child present to the emergency department with difficulty breathing. The childs parents
reports that child has history of bronchial asthma and has recently had a upper respiratory tract
infection (URI). Upon auscultation, the nurse found decresed breath sounds in the left ower lung
field. The nurse should Next assess the child’s
A. Oral temperature
B. O2 saturation
C. Apical pulse
D. Level of comforts
484. A patient with diabetic retinopathy is experiencing n episode of unresolved hemorrhage in the
eyes. The nurse identifies the most likely procedure to benefit this patient would be
A. Enuceation
Compiled by : V.Devi | 9176868900 Page 73
Diya Coaching Centre for Nurses
One candle wipes out darkness…….

B. Radial Keratotomy
C. Viterectomy
D. Peripheral iridectomy
485. A Patient admitted t the hospital with acute cholecystitis, is scheduled for surgery in the
morning, and is NPO. At 2 pm the patient develops fever of 102.4F (39.1 0 C). Medication orders
include acetaminophen 650mg orally every 4 hours as needed. The nurse should?
A. Give the medication as ordered by the physician
B. Administer the ordered dose rectally
C. Put moist cool cloth on the patients forehead and axilae
D. Notify the physician and request other orders
486. A patient is 3 weeks post operative left the below amputation. Which of the following is an
expected outcome for this patient?
A. Verbalizes relief of incision pain has intense phantom sensations
B. Participates in care plan, express concern about independence
C. Full passive range of motion, requires assistance with transfer
D. Low grade temperature, dressing reinforced every hour
487. During post operative neurovascular assessment of a patient who had a total knee amputation,
nurse assess the peronneal nerve by testing sensation?
A. On The bottom of the foot
B. In the space between the great and second toe
C. In the area anterior t rectum
D. In the anterior portion of the calf
488. The nurse is caring for a patient who is sustained atraumatic brain injury 4 days ago. The
patient remains In a pharmacologic induced comma while receiving mechanical ventilation. The
patient is on NPO status and the vital signs are within the normal range. The patient bowel sounds
are absent and nasogastric tube is connected to low intermittent suction. The nurse should prepare
to begin?
A. NG feeding
B. Rapid weaning from ventilator
C. Total parental nutrition
D. Chest physiotherapy
489. The nurse preparing to assess patients. Which of the following actions by the nurse will help to
ensure the vital signs are accurate?
A. positioning the patient supine before checking the temperature with tympanic
thermometer.
B. Placing the middle index finger over the radial artery to assess the pulse rate.
C. Positioning the hand on the chest of an infant to count the respiratory rate.
D. Placing the patient’s elbow on a support with the palm upward before checking blood
pressure
490. A 12 year old patient had a cast removed from the left leg after wearing it for 3 weeks. The
patient wants to resume sports as soon as possible. Inorder to regain the muscle strength lost while
wearing the cast , the nurse will instruct the patient to perform
A. Resistive ROM
Compiled by : V.Devi | 9176868900 Page 74
Diya Coaching Centre for Nurses
One candle wipes out darkness…….

B. Passive ROM
C. Active assistive ROM
D. Active ROM
491. During intra operative period of surgical procedure a 39 year old male has the following vital
signs. T- 98.6 F, HR -62, BP -132/78 mmhg, SP o2- 89% . The patient has received 1 Unit of PRBC and
he is in incubator. Which of the vital signs considered out of normal range?
A. Temperature
B. Oxygen saturation
C. BP
D. HR
492. A 28 year old male is recovering from moderate concussion following a motor vehicle accident
2 weeks ago. When he suddenly develops increased thrust thriving for cold water> The patient
urinates very large amount of dilute water like urine with specific gravity of 1.001-1.005. The
patient is MOST likely developing?
A. Diabetis mellitus
B. Diabetis incipidus
C. Hypothyroidism
D. Thyroid storm
493. A nurse is caring for a patient who is 6 hours post left lobectomy. On assessment the nurse
observes that the patient is becoming very restless
And nailbeds are blue and the vital signs reveals tachycardia, tachypnoea and the blood pressure is
rising. Which of the following cause is most likely?
A. Pneumonia
B. Hypoxia
C. Post operative bleeding
D. Bronco pleural fistula
494. A nurse is assessing a child with cystic fibrosis. After thoroughly assessing respiratory status
the nurse should assess which of the following?
a. The level of p[ain
B. Skin turgour
C. Clarity of urine
D. Nutrition status
495. A patient with heart failure has the following vital signs, BP-136/84mm of hg, HR-48/min, T-
98.6 F, RR-20/min. Which of these vital signs should be reported to the physician prior to
administer the next dose of Digoxin?
A. Heart rate
B. Blood pressure
C. Temperature
D. Respiratory rate
496. A nurse is caring a patient 2 hour after pacemaker placement. The patient suddenly start
complaining of chest pain and the nurse observes dyspnoea, cyanosis and absent breath sounds on
left side> Which of the following complication is anticipated?
A. Haemothorax
Compiled by : V.Devi | 9176868900 Page 75
Diya Coaching Centre for Nurses
One candle wipes out darkness…….

B. Perforation of heart
C. Pneumothorax
D. Hemorrhage
497. A nurse is preparing to administer 100m of KCl solution. The prescription indicates that this
should be infused over 2 hours. The nurse should administer howmany ml per hour?
A. 10
B. 25
C. 50
D. 100
498. While caring for a 5 year old patient with cystic fibrosis, the nurse enters the room to administer
a prescribed abbuterol nebulizer treatment. The patient is restless and anxious and the lips have a
blue gray colour. The O2 saturation is 90% what should the nurse do first?
A. Administer the nebuizer treatment as ordered
B. Administer oxygen
C. Prepare to administer CPR
D. Suction the airway
499. A community health nurse instructing a neighbourhood class about botulism (Clostridium
Botulism). The nurse teaches the group that the Most likely mode of infection would be by?
A. Direct contact with contaminated soil
B. Direct mcontact with respiratory secretions
C. Sexual intercourse
D. Injestion of contaminated food
500. A patient presents to the office for a physical. The patient is found tobe health and fit, but
occasionally drinks alcohol and has unprotected sex. What is the best nursing diagnosis/
A. Health seeking behaviour
B. Knowledge deficit high risk behaviour
C. Low sef esteem
D. Altered through processes
501. A home health nurse is preparing to administer a subcutaneous injection of heparin. When
choosing the site on the abdomen, the nurse will choose a site?
A. More than 6 inches from the umbilicus
B. More than 2 inches from the umbilicus
C. Within 1 inch of the umbilicus
D. As close as possible to umbilicus
502. A patient with pulmonary emboli complains of pain, dyspnea and a fear of dying. Which of the
following interventions would Most likely help to reduce the patients anxiety level?
A. Administer oxygen as ordered
B. Administer pain medications as ordered
C. Observe closely for signs of pain and discomfort
D. Listen to the patients concern
503. A patient with bacterial meningitis is treated with intravenous antimicrobial agent. Which of
the following Best indicates effective treatment?
A. Severe headache
Compiled by : V.Devi | 9176868900 Page 76
Diya Coaching Centre for Nurses
One candle wipes out darkness…….

B. Negative kerning’s sign


C. Nuchal rigidity
D. Photophobia
504. While caring for a patient with Pottacium deficiency the nurse would expect that the patient
may exibit?
A. Dysrhythmis
B. Olyguria
C. Diminished deep-tendon reflexes
D. Hypertension
505. A patient who underwent hand surgery requiring general anesthisia presents to the post
anesthesia care unit (PACU). After extubation the nurse should first assess?
A. Circulatory status
B. Wond status
C. Respiratory status
D. Hydration status
506. Prior to administration of an albuterol nebulizer, the nurse should help the patient assume what
position?
A. Sitting and leaning forward
B. Feet elevated above level of heart
C. High fowlers
D. Standing
507. A patient is presents to the doctor’s office 2 week status post right sided mastectomy. The nurse
needs to measure the blood pressure. Which would be the best site?
A. Above the left brachial artery
B. Right poplitiel artery
C. Above the right brachial artery
D. Left poiteal artery
508. A child with cystic fibrosis exacerbation presents to the emergency room. Which nursing
diagnosis takes First priority in planning for intervention ?
A. Imbalanced nutrition related to increased metabolic requirements because of mal
absorption
B. Defficient knowledge regarding prevention of cystic fibrosis exacerbation
C. Impaired gas exchange related to airway obstruction due to mucus
D. Interrupted family process
509. During surgery the nurse is assigned the following duties, setting up the sterile field, preparing
sutures and ligatures, assisting the surgeon during the procedure by anticipating the instruments
and supplies that will be required and labelling tissue specimen obtained during surgery. The nurse
is most likely performing what role?
A. Circulating nurse
B. Scrub nurse
C. RN first assistant
D. Nurse anesthetic

Compiled by : V.Devi | 9176868900 Page 77


Diya Coaching Centre for Nurses
One candle wipes out darkness…….

510. A nurse completes discharge instructions for patient who ws admitted 5 days ago with
pneumonia. Which statement by the patient would lert the nurse that more discharge teaching is
needed?
A. Ineed to gradually increase my activities
B. I will not need the influenza or pneumonia vaccine
C. I my experience the fatigue and weakness for a prolonged time
D. I need to have another chest x-ray in 4-6 weeks
511. The nurse is assessing a patient recently diagnosed with AIDS. Which of the following nursing
diagnosis has Priority?
A. Fear of disease progression, treatment effects, isolation and death related having AIDS.
B. Risk of infection related to immunodeficiency.
C. Ineffective brething pattern related to opportunistic infections
D. Disturbed body image related to rapid body changes from debilitating disease
512. A patient is planning to have an elective surgical procedure to repair an umbilic hernia. The
patient is 68 years of old, weight 136 kg(300lbs)and has a diabetis mellitus. Which of the following
approaches woud be the most beneficial inorder to reduce the patient’s surgical risk?
A. Monitor blood glucose level monthly
B. Avoid fluid overload by restricting fluids
C. Discourage any changes in routine before surgery
D. Encourage weight reduction
513. A nurse caring for a patient following cardiac catheterizartion evaluates the patient’s post
procedure, which of the following signs and symptoms would most likely indicate the patient
having vagal reaction?
A. Diaphoresis
B. Chest pain
C. Tingling in extremities
D. Hematoma formation
514. A home health nurse visits a patient who is newly diagnosed with diabetes. The glucose level
ranges from 120mg/dl-150mg/dl, while current Hb AIC level is 6.9%. The patient is complaint with
taking prescribed hypnoglycemic medications and eats 3 meals a day followed by desserts
sweetened with granulated sugar. The patient also exercise 30 minutes a day 3 times a week. Which
of the following education intervention take priority?
A. Glucose monitoring
B. Medications
C. Dietary requirements.
D. Exercise regimen
515. Which of the following best describes in the assessment step of the nursing process?
A. Identifying nursing interventions as appropriate for short term, immediate and long term
goal attaintment
B. Assigning priorities to the nursing priorities
C. Establishing goals of expected outcomes
D. Obtaining a nursing history and complete a physical examination of the patient

Compiled by : V.Devi | 9176868900 Page 78


Diya Coaching Centre for Nurses
One candle wipes out darkness…….

516. A nurse is measuring the chest tube drainage of a patient who had open heart surgery 4 hours
ago. Which of the following is the maximum hourly amount of chest tube drainage that is expected
in this time frame?
A. 100ml
B. 200ml
C. 300ml
D. 400ml
517. A patient report difficulty sleeping through the night since the death of spouse 6 monts ago.
Which of the following is an appropriate long term goal?
A. Feeling well rested each morning
B. Not feeling tired each afternoon
C. Taking brief naps in the middle of the day
D. Using a sleeping aid on the nightly basis
518. A patient with systemic lupus errthematous(SLE) reports decreased urinary output during the
past 2-4 days and chest pain that is aggrevated by breathing and coughing. The patients vital signs
remain within the baseline and S1 and S2 are present with audible friction rub. Which of the
following statement would be appropriate for the nurse to make ?
A. It sounds like SLE is being well controlled
B. It need to get some nytroglycerene for your chest pain
C. There may be some inflammation surrounding your heart
D. Your symptoms may be due to Urinary tract infections(UTI)
519. A patient has been hospitalised with diagnosis of Chron’s disease. The nurse can best determine
the patient’s hydration level by monitoring the:
A. Colour of urine
B. Brightness of eyes
C. Capillary refill in nail bed
D. Temperature of lower extremities
520. A patient who had abdominal surgery 6 days ago, has been ambulating the halls with out much
difficulty. However on the 7 th post operative the patient complains of increased pain at incisional
site and is walking hunched over. The most likely cause of the change is :
A. Over ssertion the day before
B. Pulmonary edema
C. Wound infection
D. Deep vein thrombosis
521. A diabetic patient come to the office for follow up 6 weeks after undergoing below the knee
amputation of the right leg for gangrene. The nurse observes that he patient is progressing well with
the use of a prosthesis and that the skin is in fact. The patient report being generally pain free wnd
itching of the right ankle. What should the nurse do/
A. Notify the doctor that there appears to be nerve damage to the right leg
B. Refer to a [pain management specialist for long term management
C. Refer to Psychiatrist for evaluation since the patient hs no right ankle.
D. Explain the phenomena of phantom pain and phantom sensation to the patient

Compiled by : V.Devi | 9176868900 Page 79


Diya Coaching Centre for Nurses
One candle wipes out darkness…….

522. A 1 year old child presents at the clinic one week after hospitalisation for surgical repair of a
fractured right femur. The patient is receiving pain medications every morning and evening. The
best way to evaluate the effectiveness of the pain management plan:
A. To ask child in simple terms about the comfort level over the past weeks
B. By direct observation of the child’s non verbal behaviour during the visit.
C. To teach the child how to use the Wong/Baker Faces ain rating scale
D. To interview the parent about behaviour moods and sleep pattern over the past weeks
523. A patient visits the clinic for first time. In order to perform an accurate and complete
assessment, which of the following would be the nurse’s first step?
A. Obtain temperature, pulse and respiration
B. Obtain a complete history
C. Obtain a blood pressure
D. Perform a review of systems
524. The nurse is assigned to care for patients who have recently been diagnosed with Crohns’s
disease. The best treatment is usually?
A. Dietary changes
B. Reversible colostomy
C. Permanent colostomy
D. Watchful waiting
525. A Patient comes to the medical office with complains of some urinary incontinence. The nurse
discovers the incontinence occurs because of an inability delay voiding long enough to reach a toilet
after the patient feels a sensation of bladder fullness.This type of incontinence is:
A. Stress
B, Urge
C. Overflow
D. Functional
526. When caring for chid with spina-bifuda, the nurse knows that the child has increased risk for
allergy to:
A. Peanuts
B. Straw berrys
C. Eggs
D. Latex
527. A chid has been admitted with third degree burns covering approximately 8%..... of the body.
Which of the following nursing diagnosis takes First priority?
A. Imbalance nutrition related to hyper metabolic state
B. Disturbed body image related to patients perception of altered appearances.
C. Impared skin integrity related to burn injury
D. Deficient fluid volume related to fluid shift
528. While planning a class on pregnancy the nurse should include symptoms of pregnancy that
must be reported immediately such as:
A. Leg cramps
B. Visual disturbances
C. Swelling of the legs
Compiled by : V.Devi | 9176868900 Page 80
Diya Coaching Centre for Nurses
One candle wipes out darkness…….

D. Constipations
529. Which of the following reacts to viruses and bacteria by increasing number?
A. Antigen
B. Antibodies
C. Rh factor
D. Platelets
530. Which nursing diagnosis takes priority for newly diagnosed patient with a sided stroke?
AS. Risk of impaired swallowing related to absent gag reflex
B. Risk for impaired skin integrity related to immobility
C. Risk for infection related to invasive line placement
D. Risk for impaired speech related to left side stroke
531. A nurse taking care of a patient who underwent abdominal surgery 3 years ago. The patient
has not been able to breath deeply and refuses to get out of bed since surgery due to pain. Also the
patient complaints of shortness of breath and the sounds are diminished upon auscultation.
The vital signs are BP; 120/70mm of hg. HR-75, RR-22, T-36.40c (97.6F),> which of the following
should the nurse suspected?
A. Sepsis
B. Atelectasis
C. CHF
D. Emphysema
532. The nurse assessing a 15 year old patient who is being admitted due to an excacerb ation of
bronchial asthma. The nurse should give priority to tasking if the patient has a history of:
A. Indoor allergies
B. Intubation
C. Chest trauma
D. Coxsackle virus
533. A community health visits a patient who had a right foot amputation> Which of the following
would suggest that the patient is meeting the expected outcome for this type surgery?
A. Stays in bed
B. Verbalize constant pain
C. Avoid social gathering
D. Accepts altered body image
534. While reviewing stress management techniques with a patient diagnosed with multiple
Sclerosis. What would the nurse identify as Most appropriate ?
A. Relaxation in warm bubble bath
B. Yoga in a cool rom
C. Sun bathing
D. Cross country running
535. A nurse visits the home of a patient who is one week post left breast mastectomy. Which of the
following should be included in patient education?
A. It is OK to use a straight edge razor when shaving
B. Blood pressure checks should be done in the left arm
C. Cuticle should not be cut
Compiled by : V.Devi | 9176868900 Page 81
Diya Coaching Centre for Nurses
One candle wipes out darkness…….

D. Avoid insect repellent on the left arm


536. A Patient is 24 hours post operative after having a right total hip arthroplasty. The patient
complaints of the pain in the right calf rated 6 on a scale 0 (no pain) to 10(severe). The nurse
observes that the right calf is warm and tender to touch, which recognizes that they are classical sign
of :
A. Ineffective tissue perfusion
B. Fluid overload
C. Arterial occlusion
D. Deep vein thrombosis
537. A patient with dementia treated for dehydration. The patient is confused and has been
immobile for the past month. Currently the patient is incontinent and unable to feed self. The
nursing care plan would include:
A. Coughing and deep breathing exercises
B. Positioning and turning every 2 hours
C. Range of emotion exercises to all extremities every hour
D.Ambutate at least 20 steps every shift
538. A chid comes in to the clinic with severa lesions on the scalp. The rund lesions have dandruff
like scaling with hair loss. What is the most likely diagnosis for this skin condition?
A. Impetigo
B. Ring worm (tinea capitis)
C. Pediculosis capitis
D. Scabies
539. A nurse evaluate a patient for signs of rebleeding from ruptured intracranial aneurysm that
required surgical ligation. The highest risk for aneurysm rebleed is within?
A. 6 hours
B. 24 hours
C. 48 hours
D. 72 hours
540. When discussing dietary choices of a patient who is receiving heparin therapy, the nurse should
stress that which of the following foods affect the cloting time?
A. High protein diet
B. Soy-based foods
C. Food high in Vitamin K
D. Foods containing Cows milk.
541. A patient admitted to the hospital for pneumonia, finishes a course of levofloxacin(L vaquin)
lungs are clear and the patient is no longer coughing. Which of the following post discharge
laboratory results best indicates desired outcomes?
A. Normal WBC
B. Normal haematocrit count
C. Normal platelet count
D. Normal potassium

Compiled by : V.Devi | 9176868900 Page 82


Diya Coaching Centre for Nurses
One candle wipes out darkness…….

542. A home health care nurse visits a patient diagnosed with rheumatic. The nurse gathers
information about the pain level after the use of the prescribed pain medication to check the
effectiveness of the intervention. This phase of nursing process is called?
A. Assessment
B. Diagnosis
C. Implementation
D. Evaluation
543. A plan of care for a child with cerebral palsy should include all of the following Expect?
A. Physical therapy
B. Play
C. Feeding
D. Bowel and bladder training
544. A patient is admitted to the hospital with Klebsiella pneumonia. During the initial intravenous
dose of amikin(amikacin sulphate) the patient develop severe respiratory distress. This is most
likely?
A. A side effect
B. An indication of drug tolerance
C. A drug allergy
D. A toxic effect
545. A patient is diagnosed with peptic ulcer> What would be the long term goal for this patient?
A. Patient remains free from the signs and symptoms of gastro intestinal bleeding
B. Patient maintains ife style alterations to prevent recurrence of ulcer.
C. Patient expresses decreased pain level
D. Patient performs activities of daily living without difficulty
546. A patient experienced cerebrovascular accident (CVA) one week ago. The doctor orders the
patient discharged from the hospital to home health care. The visiting nurse evaluates the patient
and notes, the patient has difficulty initiating speech and forming words. Which of the following
type of aphasia is most likely is this?
A. Receptive
B. Expressive
C. Fluent
D. Conduction
547. A Nurse is evaluating the home of patient with left sided paralysis. Which of the following
observation would indicate that the patient is complying with home-based safety?
A. The telephone is on the bed side table which is next to the head of the bed.
B. The bedside commode is on the left side of the bed with back of the commode facing the
foot of the bed.
C. The walker has wheels on its back legs and has tennis balls on the front legs
D. The stair leading from the bedroom to the living area a handrail on the right side of the
stairway.
548. A patient is admitted to the hospital with a CVA; right hemipegia and expressive aphasia. With
a nursing diagnosis of impared verbal communication, what is the Best short term goal for this
patient?
Compiled by : V.Devi | 9176868900 Page 83
Diya Coaching Centre for Nurses
One candle wipes out darkness…….

A. Learn to speak clearly in 30 days


B. Communicate effectively within one week
C. Have al needs anticipated by staff daily
D. Make basic needs known daily
549. A patient has a dissection aneurysm. The patient’s surgery would be categorized as:
A. Elective
B. Urgent
C. Emergency
D. Diagnostic
550. A parents to the emergency room due to overdose of morphine sulphate. Which of the
following should the nurse have readily available?
A. Glucogen
B. Antibiotic
C. Acetycysteine
D. Nalaxone (narcan)
551. A patient with iron deficiency anaemia due to an insufficient iron intake need to learn to select
better food choices. The nurse works with this patient to establish a plan of care and provide
education on proper nutrition and good source of iron. Besides educating the patient on a well
balanced diet the nurse would Most likely teach the patient that good sources of iron include:
A. Seafood, cheese, soya bean oil and chocolate
B. Animal protein, egg yolk ,dried fruits and nuts
C. Dairy product, citrus fruits, fish liver oils and poultry products
D. Sea food, fruit, poultry and tomatoes
552. A 42 year old patient is in lower body cast following a motor vehicle accident. In order to
minimise muscle strength loss while in the cast, the nurse will instruct the patient in the
performance of
A. Isometric exercises
B. Passive range of motion exercises
C. Active range of motion exercises
D. Resistive range of motion exercises
553. A patient is being followed in the clinic for hypertension, adult onset diabetes and obesity. The
patient is apathetic about learning nutritional guidelines to reach the goals of weight loss and
consumption of a healthy diet. The patient admitted to eating “whatever is put in front of me”.
Which of the following actions would the nurse take?
A. Colaborrate with the patient to set goals
B. Add a nursing diagnosis of non compliance
C. Refer for psychiatric screening for depression
D. Discuss nutritional intervention with the spouse
554. A child is admitted to the paediatric ward with fever, leathery joint pain and abdominal pain
for several weeks. The patient has a history of recurrent respiratory and ear infections. Physical
findings include widespread ecchymosis, generalized lymphadenopathy, hepatomegaly and pallor.
Lab works show a low haemoglobin level, low RBC evel, low haematocrit and platelets. The nurse
should expect the bone marrow stain to show?
Compiled by : V.Devi | 9176868900 Page 84
Diya Coaching Centre for Nurses
One candle wipes out darkness…….

A. large number of lymphoplasts and lymphocytes


B. Low number of lymphoplasts and large number of lymphocytes
C. Low number of lymphopasts and lymphocytes
D. Large number of lymphoplasts and low number of lymphocytes
555. Immediately following the birth of a full term newborn, which of the following nursing
diagnosis should tke priority?
A. Ineffective airway clearance related to nasal and oral secretions
B. Ineffective thermoregulation related to environmental factors
C. Risk for imbalanced fluid volume related to weak sucking reflex
D. Risk for injury related to immature defence mechanism
556. A patient receives intravenous therapy of 1000cc normal saline with 20 MEq potassium chloride
at a rate of 75cc per hour. Upon evaluation of the site there is no edema, the vein appears slightly
red and the patient complaints of pain. Wht should the nurse do/
A. Slow the rate to prevent burning from the solution and continue to monitor
B. Discontinue the intravenous line and restart in another site
C. Monitor at least every half hour for edema but continue as order state.
D. Notify the doctor that the patient is having an adverse reaction to themediction
557.A healthy 26 year old patient is at 39 weeks gestation. The patient is not considered high risk at
the time of admission to the labour and delivery unit. Which of the following pending laboratory
test results should receive priority?
A. RBC count
B. Haematocrit
C. WBC count
D. Blood type
558. A patient comes to the emergency department with extreme dyspnea, orthopnea anxiety and
complaints of feeling panicky. The patient is coughing up frothy sputum and is cyanotic with
profuse perspiration. Inspiratory and expiratory wheezing and hubbling sounds are auscultated.
The patient is diagnosed with acute pulmonary edema. What should the nurse do first?
A. Identify precipitating factors and underlying conditions
B. Administering morphein (Duramorph) to reduce anxiety
C. Assess oxygen saturation rate.
D. Administer digoxin (lanoxin) to decrese fluid backing up into the lungs
559. During surgery the patient has the following intake and output; Intravenous fluids -650cc,
intravenous antibiotic-50cc; 1 Unit of red blood cels (PRBC)-350cc; Nsogastric output-120cc,
Estimated blood loss 80cc. And urine in the foley’s catheter is 240cc. What is the patients total
output?
A. 120cc
B. 200cc
C. 240cc
D. 440cc
560. A 25 year old female presents to the emergency department room with leathary, decreased
reflexes, hypoventilation, hypotension and fixed dilated pupils. A family member who is
accompanying the patient has an empty bottle of diazepam (valium) which the label states was
Compiled by : V.Devi | 9176868900 Page 85
Diya Coaching Centre for Nurses
One candle wipes out darkness…….

recently refilled. The family member also indicates that the patient has a history of depression. What
intervention should the nurse expect to administer?
A. Nalaxone (Narcan)
B. Activated charcoal
C. Magnecium sulphate to reduce the risk of seizure
D. Tap water enema
561. A nurse is signed to acre for patient with a diagnosis pf thrombotic stroke. The nurse knows
that this type of stroke is most likely caused by?
A. Blockage of large vessels as a result of artherosclerosis
B. Emboli produced from vascular heart disease
C. Decreased cerebral blood flow due to circulatory failure
D. A temporary disruption of oxygenation of the brain
562. The nurse administered a prescribed intra muscular medication to a patient during a home
visit. How should the nurse dispose the used needle and syringe?
A. Recap the needle, then place the needle and syringe into a aterproof continer until sfe
disposal can be made.
B. Bend the needle back towards the barrel of the syringe before putting the needle and
syringe in a met trash container
C. wrap the needle and syringe in disposable paper before putting the needle and syringe
into the dirty section of the nurses equipment bag
D. Put the needle and syringe directly into a puncture resistant plastic container that has a
lid
563. What is the most common characteristic of stage 4 pressure ulcer?
A. Pink skin
B. Presence of sinus tracts
C. Exposure of bones
D. Infection
564. While visiting a patient with a new colostomy, the home care nurse observes that the skin
around the stoma cite is red. Which inventions should the nurse do next?
A. Apply pectin, gelatine, or synthetic skin barrier around the stoma.
B. Apply triple antibiotic to the raw skin and leave it open to the air
C. Instruct to empty the pouch as soon as stool is present
D. Instruct to remove the bag and skin barrier after each stool
565. A nurse educates a patient about the use of incentive spiro metry to prevent atelectasis after a
surgery. The nurse is performing what step of nursing process?
A. Diagnosis
B. Assessment
C. Implementation
D. Evaluation
566. A patient had suicidal tendency. He was very depressed and inactive. Doctor prescribed
SSRI and he took this medicine around 1 month. Now he is very active and energetic but suicidal
tendency is still with him. What is his grade of suicidal tendency?
A, None
Compiled by : V.Devi | 9176868900 Page 86
Diya Coaching Centre for Nurses
One candle wipes out darkness…….

B. Mild
C. Moderate
D. Severe
567. A patient likes to tke enema himself. Which position is most suitable?
A. Sims
B. Left lateral
C. Trendelenberg
D. Prone
568. Cardiac surgery position?
A. Trendelenberg
B. Supine
C. Dorsal recumbent
D. Semi fowler’s
569. A patient is on heparin therapy and the patient continues to bleed during therapy. Which
medication should be administered to prevent it?
A. Morphine
B. Vit.K
C. Nalanone
D. Flumazenil
570. What is the priority action of a nurse for a patient with following vitas emergency room?
Pulse -86/mt
Respiratory rate -22/mt
Blood pressure -100/60 mm of hg
Temperature -390C
A. Normalize the temperature
B. Continue to monitor vitals
C. Inform the doctor
D. Document the event
571. A patient comes to the emergency department with cyanosis and edema. Which position to be
given?
A. Trendelenburg
B. Head elevated position
C. Semi flower position
D. Sims position
572. A patient has undergone surgical removal of parathyroid gland. After 2 weeks the patient left a
tingling sensation, numbness in the extremities. The nurse should prescribe which type of food
products?
A. Cheese, orange, meat
B. Whole milk, Grains, fish
C. Green leafy vegetables
D. Fruit juices

Compiled by : V.Devi | 9176868900 Page 87


Diya Coaching Centre for Nurses
One candle wipes out darkness…….

573. A patient has tenderness on the right upper quadrant and pain on the abdomen. Abdominal
distention is present on prescription dull sound can be heard. On the patients right side also bluish
discolouration on the umbilical region. Which may be the condition?
A. Hepatomegly
B. Hiatal hernia
C. Gstric ulcer
D. Ascitis
574. For a patient having seizure, which management goes to priority?
A. Side lateral position
B. O2 administration
C. Restrain the patient
D. Use mouth pieces
575. A patient is on heparin treatment started warfarin. For this patient which lab test should be
done?
A. APTT
B. Bilirubin level
C. PTT INR
D. Ph of the blood
576. A chid with type1 Diabetes mellitus has taken morning dose insulin and went for sport
activities. The child suddenly became cool and confused. What is the immediate management?
A. Provide orange juice 1 glass
B. 5% dent rose bolus administration
C. Provides 2 mg glucagon
D. Provide candy to the child
577. A child with attention deficit has brought to a primary health nurse. The teachers tod that the
child is very restless and not attentive in the class. The best suggestion by the nurse is
A. Make the child to follow strict schedule
B. Make him to sit with a grade children
C. Make him to set at last bench
D. Place the child in a less stimulated class
578. A patient fall from the bed. The nurse makes him to sit on the bed. The patient has rapid
swallow breathing, shortening of limb, profuse sweating and pain on the leg. Which management
gets the priority?
A. Pain management
B. O2 administration
C. Immobilisation of affected leg
D. Check V/s
579. Precaution for ‘Diphtheria’
A. Droplet
B. Airborne
C. Contact
D. Reverse isolation

Compiled by : V.Devi | 9176868900 Page 88


Diya Coaching Centre for Nurses
One candle wipes out darkness…….

580. An obese patient, who is not interested to mobilize properly, has calf muscle swelling , pain and
cannot even raise the leg. Which drug can be infused to treat this condition?
A. heparin injection
B. Hot, warm application on the leg
C. Compression stockings
D. Immobilize the leg
581. A post op patient requested for pain relief, the nurse has administered morphiene 5 mg
according to the order. After 10 minutes the patient become confused decreased respiration rate and
with no response.
BP 100/70mm of Hg
HR 110/mt
RR 8/mt
SPO2 88 %
Temp 36.40C
What is the priority management?
A. Administer Oxygen
B. Barium enema
C. Administer nalaxone
D. Administer IV fluids
582. A child came with the complaints of nausea, vomiting, bloody stool and mucus.
Which laboratory test should be assessed?
A. Hb count
B. Barium enema C. Urine list
D. Fecal occult blood test
583. A patient came to the hospital with a abdominal pain and nausea. Diagnosed on having stone
in the bile duct. Which assessment can consume this?
A. Dark and tarry stool
B. Amber colour urine
C. Yellowish sclera
D. Edema on Extremities

Compiled by : V.Devi | 9176868900 Page 89

Das könnte Ihnen auch gefallen